Head and Neck Congenital 01-10, 18 Flashcards

1
Q

A 6-month-old infant is brought to the office because of abnormal head tilt and limited range of motion of the neck. Physical examination shows a 3-cm firm mass on the right side of the neck above the clavicle. Which of the following physical findings is most likely associated with this condition?

A) Flexion of the head toward the left shoulder
B) Flexion of the neck toward the left shoulder
C) Limited lateral flexion toward the left shoulder
D) Limited lateral flexion toward the right shoulder
E) Rotation of the head to the right shoulder

A

The correct response is Option C.

Torticollis is a congenital neck deformity involving shortening of the sternocleidomastoid muscle. The symptoms are head tilt and limited range of motion in the neck, and there is usually a firm mass in the body of the sternocleidomastoid muscle. The majority of the symptoms resolve in the first year of life. Occasionally, large portions of the muscle may become fibrotic, necessitating surgical release.

On physical examination, findings include flexion of the head and neck toward the ipsilateral shoulder, rotation of the head and neck to the contralateral shoulder, and a lack of lateral flexion toward the contralateral shoulder. In the case of right-sided torticollis, the findings would be:

Flexion of the head and neck toward the right shoulder

Rotation of the head and neck toward the left shoulder

Limited lateral flexion toward the left shoulder

How well did you know this?
1
Not at all
2
3
4
5
Perfectly
2
Q

A 45-year-old man comes to the office for follow-up examination 3 months after undergoing surgical repair of a through-and-through laceration of the left cheek. During the procedure, an injury to the parotid duct was noted and repaired. He says he feels fine, but he now has difficulty playing the trumpet because he is unable to create sufficient air pressure in his mouth. Which of the following muscles was most likely also severed?

A ) Buccinator

B ) Levator labii superioris

C ) Masseter

D ) Risorius

E ) Zygomaticus major

A

The correct response is Option A.

The buccinator muscle is the only muscle of facial expression that compresses the cheeks, which is an essential function for playing air-based instruments such as the trumpet. Both the buccinator and the orbicularis oris compress the lips, also necessary for playing trumpets. The buccinator muscle ordinarily contributes to the function of forming a food bolus during mastication. The path of the parotid duct typically leaves the parotid gland from its anterior border and courses superficially to the masseter muscle toward the mid cheek and then pierces the substance of the buccinator muscle, reaching the mucosa of the oral cavity opposite the maxillary second molar.

The levator labii superioris, the risorius, and the zygomaticus major muscles all have a function that contributes to separating the lips, which releases the pressure from inside the mouth. These muscles arise from bone and fascia and attach to the lips. The masseter, which originates in the zygomatic arch and inserts on the lateral surface of the ramus, elevates the mandible. The masseter has no role in holding intrabuccal or lip pressure.

How well did you know this?
1
Not at all
2
3
4
5
Perfectly
3
Q

A 20-year-old man comes to the office because he has had paraesthesia of the anterior lateral aspect of the tongue since undergoing removal of the mandibular third molars 3 weeks ago. The most likely cause is injury to which of the following nervous structures?

A ) Chorda tympani

B ) Facial

C ) Glossopharyngeal

D ) Hypoglossal

E ) Lingual

A

The correct response is Option E.

General sensation of the anterior two thirds of the tongue is supplied by the lingual nerve, which is a branch of the mandibular division of the trigeminal. Taste in the anterior two thirds of the tongue is supplied by the chorda tympani from the facial nerve. The chorda tympani joins the lingual nerve and runs anteriorly in its sheath. The glossopharyngeal nerve supplies the mucosa of the posterior one third of the tongue. The hypoglossal nerve is the motor nerve to the tongue, and the facial nerve is the motor nerve to the face.

How well did you know this?
1
Not at all
2
3
4
5
Perfectly
4
Q

A 26-year-old woman is being evaluated because she has had complete left-sided, flaccid facial paralysis since she awoke 3 hours ago. She also has a metallic taste in her mouth and hypersensitivity to sound. Denervation of which of the following muscles is the most likely cause of the hypersensitivity to sound?

A ) Levator palatini

B ) Stapedius

C ) Tensor tympani

D ) Tensor veli palatini

E ) Zygomaticus major

A

The correct response is Option B.

The 26-year-old woman described has the typical history of Bell palsy. In patients with Bell palsy, the entire nerve is inflamed, but the maximum conduction block is either in the meatal or labyrinthine segments. Because the conduction block is proximal to the chorda tympani and stapedial nerve, patients also experience a change in taste and a decreased ability to accommodate (ie, dampen) loud noises.

When an acoustic tumor causes facial paralysis, the paralysis is usually slowly progressive. Patients generally present with a hearing loss, not a hypersensitivity to noise.

The tensor tympani muscle attaches to the malleus and is innervated by the trigeminal nerve, not the facial nerve. The tensor veli palatini muscle is also innervated by the trigeminal nerve and is responsible for active dilatation of the eustachian tube. Blockage of the eustachian tube would cause a hearing loss.

The zygomaticus major muscle is innervated by the facial nerve. Denervation causes a decreased ability to smile, not hearing loss. The levator palatini muscle is innervated by the vagus nerve.

How well did you know this?
1
Not at all
2
3
4
5
Perfectly
5
Q

A 58-year-old man comes to the office for consultation regarding treatment 3 weeks after receiving a diagnosis of squamous cell cancer of the soft palate. He says he has had pain in the left ear for the past 2 months. Examination of the ear shows no abnormalities. The most likely cause of the pain is involvement of which of the following nerves?

A ) Auricular branch of the vagus (X)

B ) Auriculotemporal

C ) Great auricular

D ) Superficial temporal

E ) Vestibulocochlear (VIII)

A

The correct response is Option A.

The auricular branch of the vagus nerve (Arnold nerve) carries sensory input from the ipsilateral concha and oropharynx. Chronic external ear pain may alert the astute physician of more serious intraoral pathology. The auriculotemporal nerve carries sensory information from the anterior and superior external auditory canal, and the great auricular nerve relays sensation from the lower half of the ear. The lesser occipital nerve provides sensory input from the superior cranial surface of the ear. Although the superficial temporal nerve is not an anatomically recognized structure, the superficial temporal artery supplies vascular inflow to the anterior external ear.

How well did you know this?
1
Not at all
2
3
4
5
Perfectly
6
Q

A 45-year-old woman comes to the office because she has had a 4-week history of a rapidly enlarging left parotid mass. On physical examination, the patient has ptosis of the left eyebrow and is unable to fully close the left eye or depress the left lower lip. The most likely cause of these findings is tumor involvement at which of the following foramina at the base of the skull?

A ) Jugular

B ) Lacerum

C ) Ovale

D ) Rotundum

E ) Stylomastoid

A

The correct response is Option E.

A history of a progressive facial paralysis associated with a parotid mass suggests the diagnosis of a malignant parotid tumor. The temporal, zygomatic/buccal, and ramus mandibularis branches are affected, indicating that the main trunk of the facial nerve is invaded by tumor. The facial nerve exits the skull base from the stylomastoid foramen. The foramen lacerum, foramen ovale, and the foramen rotundum contain the internal carotid artery, mandibular (V3) nerve, and maxillary (V2) nerve, respectively. The glossopharyngeal (IX), vagus (X), and spinal accessory (XI) nerves emerge from the jugular foramen.

How well did you know this?
1
Not at all
2
3
4
5
Perfectly
7
Q

A 5-year-old boy is brought to the office because of a 10-day history of inflammation of a midline neck mass that his parents first noticed 1 year ago. Physical examination shows a 35-mm mass just inferior to the hyoid bone. Which of the following is the most likely diagnosis?

A ) Branchial cleft remnant

B ) Lingual thyroid gland

C ) Mucoepidermoid carcinoma

D ) Reactive lymph node

E ) Thyroglossal duct cyst

A

The correct response is Option E.

The patient described has a thyroglossal duct cyst. Thyroglossal duct cysts can form anywhere along the thyroglossal duct, which extends from the foramen cecum of the tongue to the final position of the thyroid gland in the neck, below the laryngeal cartilage. Normally, the thyroglossal duct atrophies and disappears. However, a remnant of it may persist and form a cyst in the tongue or anterior midline of the neck, most commonly inferior to the hyoid bone. Thyroglossal duct cysts are often asymptomatic unless they become infected, as this one has.

Reactive lymph nodes are the most common neck mass in children. They are usually found laterally in the submandibular and jugulodigastric areas. Branchial cleft remnants (sinuses and cysts) arise from the branchial apparatus. They are also located laterally, along the anterior border of the sternocleidomastoid muscle, usually just inferior to the angle of the mandible. Lingual thyroid glands are a type of ectopic thyroid located within the tongue. Ectopic thyroid glands can be located along the course of the thyroglossal duct and are a result of failure of the thyroid to descend. Unlike thyroglossal duct cysts, they represent the only thyroid tissue present in the patient. Mucoepidermoid carcinomas are salivary gland malignancies found in children, and they most commonly appear within the parotid gland.

How well did you know this?
1
Not at all
2
3
4
5
Perfectly
8
Q

A 4-year-old child has a congenital sinus tract opening at the anterior border of the lower third of the sternocleidomastoid muscle. Which of the following nerves is most likely to be injured during surgical excision of the fistulous tract?

A ) Accessory (XI)

B ) Facial (VII)

C ) Hypoglossal (XII)

D ) Lingual

E ) Vagus (X)

A

The correct response is Option C.

The child described has a branchial cleft sinus or fistula, which is an embryologic remnant of the cleft between the second and third branchial arches. The fistulas can be bilateral in up to 30% of cases. Histologically, they are lined with stratified squamous epithelium but may also have some pseudostratified columnar ciliated epithelium in their walls. They also contain keratin, hair follicles, sweat glands, and sebaceous glands. They must be completely excised to prevent recurrence.

The second branchial arch descends over the third, resulting in an external opening in the lower neck. The internal opening lies at the anterior aspect of the posterior pillar of the fauces, just behind the tonsil (which is the junction between the second and third branchial arches). Usually, the fistula will follow the carotid sheath upwards before crossing the hypoglossal (XII) nerve and passing between the internal and external carotid arteries to reach the tonsillar fossa. As a result, the hypoglossal nerve is at risk during surgery. The facial (VII) nerve is not in the vicinity of the fistula. The facial nerve is intimately related to first branchial arch sinuses and cysts. The spinal accessory (XI) nerve is in the posterior triangle of the neck and is not in the vicinity of the surgical field to remove this type of fistula. The lingual nerve lies at a higher level and is safe. The left recurrent laryngeal nerve arises from the vagus (X) nerve low in the neck before hooking around the subclavian artery and passing medially behind the common carotid artery to reach the groove between the trachea and the esophagus. As a result of this anatomy, it too is out of harm €™s way. The vagus (X) nerve lies in the carotid sheath behind, and somewhat between, the internal jugular vein and the common carotid artery. The fistula lies on the other side of the great vessels. At the carotid bifurcation, the vagus nerve is lateral to the fistulous tract and stands less chance of injury than does the hypoglossal nerve.

How well did you know this?
1
Not at all
2
3
4
5
Perfectly
9
Q

A 20-year-old man comes to the emergency department because of a deep laceration of the lower lip. Bilateral mental nerve blocks will be used to anesthetize the lip properly prior to repair. Which of the following is the most appropriate landmark for needle placement for the blocks?

A ) Mandibular second premolar

B ) Maxillary canine

C ) Oral commissure

D ) Retromolar fossa

E ) Sigmoid notch

A

The correct response is Option A.

The inferior alveolar nerve enters the mandible on the medial side of the ramus about 10 mm below the sigmoid notch. It then courses through the canal closest to the buccal cortical plate in the region of the ramus, angle, and down to the third molar with an average distance of 1.8 mm ± 1 mm. The nerve then swerves away at a position of 4.1 mm + 1 mm from the buccal cortex as it passes the region of the first and second molars. As it traverses the mandibular body, it is lowest and closest to the inferior cortex (7.5 + 1.5 mm) near its exit site at the level of the first molar and second premolar via the mental foramen on the anterior surface of the mandible. The mental nerve supplies the skin of the lower lip and chin right up to the midline.

The maxillary canine may be used as a landmark for needle insertion toward the infraorbital foramen during infiltration of the infraorbital nerve. The maxillary second molar is a landmark typically used to locate the opening of the Stensen duct.

The oral commissure is used for facial aesthetic measurements and not for nerve blocks.

The retromolar fossa, posterior to the mandibular third molar, is the preferred location for needle insertion to anesthetize the buccal nerve, which normally supplies sensibility to the central cheek.

The sigmoid notch is used as a landmark to reference the location of the inferior alveolar nerve.

How well did you know this?
1
Not at all
2
3
4
5
Perfectly
10
Q

In the temporal region, the frontal branch of the facial nerve is located within which of the following layers?

A ) Subcutaneous tissue

B ) Superficial temporal fascia

C ) Superficial layer of the deep temporal fascia

D ) Superficial temporal fat pad

E ) Deep layer of the deep temporal fascia

A

The correct response is Option B.

As depicted in the image shown, the temporal or frontal branch of the facial nerve is found within the superficial temporal or temporoparietal fascia. In the temporal region, the facial nerve can be injured with a coronal approach. The layers from superficial to deep in this region include: (1) skin, (2) subcutaneous tissue, (3) superficial temporal fascia also known as the temporoparietal fascia, (4) superficial layer of the deep temporal fascia, (5) superficial temporal fat pad, (6) deep layer of the deep temporal fascia, (7) temporalis muscle.

When the coronal flap is raised, as soon as the yellow superficial temporal fat pad is seen beneath the superficial layer of the deep temporal fascia, the superficial layer of the deep temporal fascia must be incised and included with the coronal flap to protect the frontal branch, which is in the superficial temporal fascia (temporoparietal fascia), one layer superficial to this.

How well did you know this?
1
Not at all
2
3
4
5
Perfectly
11
Q

The optic nerve passes through which of the following bones of the orbit?

A ) Ethmoid

B ) Frontal

C ) Lacrimal

D ) Maxilla

E ) Sphenoid

A

The correct response is Option E.

Seven bones make up the orbit: the frontal bone, maxilla, zygoma, ethmoid, lacrimal, greater and lesser wings of the sphenoid, and the palatine bone. The frontal, maxilla, zygoma, and ethmoid bones constitute the strong outer rim of the orbit and protect the more delicate bones in the interior orbit. The lesser wing of the sphenoid forms the posterior aspect of the roof of the orbit and transmits the optic nerve and ophthalmic artery through the optic canal. The greater wing of the sphenoid contains the superior orbital fissure, which transmits the lacrimal nerve, frontal nerve, trochlear nerve, superior and inferior branches of the oculomotor nerve, the nasociliary nerve, and the abducens nerve.

How well did you know this?
1
Not at all
2
3
4
5
Perfectly
12
Q

The levator veli palatini muscle is innervated by which of the following nerves?

A ) Facial (VII)

B ) Hypoglossal (XII)

C ) Mandibular branch of the trigeminal (V)

D ) Maxillary branch of the trigeminal (V)

E ) Vagus (X)

A

The correct response is Option E.

The levator veli palatini (levator muscle) is a muscular sling in the posterior palate that is critical for palatal closure. Embryologically, the muscle arises from the fourth pharyngeal arch and, as a result, is innervated by the pharyngeal plexus, a derivative of the vagus (X) nerve. The two portions of the levator muscle arise from the eustachian tube and sphenoid bone while inserting into the other levator muscle in the midline of the soft palate. The function of the levator muscle is to elevate and posteriorly reposition the soft palate against the posterior nasal wall to assist in speech. This closes the velopharyngeal port while opening the eustachian tubes. The vagus also innervates several other muscles involved in normal speech development: the palatoglossus, the pharyngeal constrictors, the musculus uvulae, and the palatopharyngeus. The palatoglossus muscles form the anterior tonsillar pillars and lift the base of the tongue. The musculus uvulae originate from the palatine aponeurosis and the posterior nasal spine and insert into the uvula. With speech, these muscles will contract, causing a bulging of the soft palate and subsequent velopharyngeal closure. The uvula is lifted and bent backward.

The facial (VII) nerve innervates the muscles of facial animation, including the buccinator.

The hypoglossal (XII) nerve provides motor function to the tongue.

The trigeminal (V) nerve has three individual branches. The first branch, the ophthalmic division, is strictly a sensory branch. The second, or maxillary, division is also sensory. The third branch, the mandibular division, is the largest branch and is a mixed sensory and motor branch. Embryologically, the mandibular division arises from the first branchial (or mandibular) arch and provides motor function to the muscles arising from this arch. These muscles include the temporal, masseter, pterygoids, mylohyoid, tensor tympani, and the anterior belly of the digastric. It also innervates the tensor veli palatini. This muscle arises from the eustachian tube and medial pterygoid plate, wraps around the hamulus, and then inserts into the midline of the soft palate. During swallowing, these muscles tense the soft palate, opening the eustachian tube and allowing the tongue to move food posteriorly.

How well did you know this?
1
Not at all
2
3
4
5
Perfectly
13
Q

A 35-year-old man is brought to the emergency department following a motorcycle collision. Examination shows swelling and deformity of the mandible. Radiographs show a left subcondylar fracture with the proximal fragment displaced forward. Which of the following muscles is most likely involved in the displacement?

A ) Digastric

B ) Lateral pterygoid

C ) Masseter

D ) Medial pterygoid

E ) Temporalis

A

The correct response is Option B.

The origin of the lateral pterygoid muscle (shown below) has two components: one that attaches to the temporomandibular joint capsule and disc, and one that attaches to the fovea in the neck of the mandible. Proximally (anteriorly), the superior head attaches to the greater wing of the sphenoid bone, and the inferior head to the lateral surface of the lateral pterygoid plate. Therefore, in a subcondylar fracture, the lateral pterygoid muscle is in position to draw the proximal fragment forward.

The digastric muscle extends from the mastoid process of the temporal bone to the hyoid bone and to the anterior mandible. It depresses the mandible (inferior movement) or elevates the hyoid. The masseter, medial pterygoid, and temporalis elevate the mandible from the region of the ramus, angle, or coronoid process. The masseter originates in the zygomatic arch and inserts on the lateral surface of the ramus. The medial pterygoid originates on the lateral pterygoid plate and the tuberosity of the maxilla and attaches to the medial surface of the angle of the mandible. The temporalis muscle is fan-shaped and originates from the floor of the temporal fossa and inserts in the coronoid process and anterior border of the ramus.

How well did you know this?
1
Not at all
2
3
4
5
Perfectly
14
Q

The nasolacrimal duct drains into the nose at which of the following anatomic locations?

A ) Inferior concha

B ) Inferior meatus

C ) Middle concha

D ) Middle meatus

E ) Superior concha

A

The correct response is Option B.

The nasolacrimal duct, which maintains the drainage of the nasolacrimal system, drains into the inferior meatus just below the inferior concha (see diagram). Other structures also drain into the nose. The auditory tube opens into the nasopharynx just posterior to the inferior concha. The sphenoid sinus drains into the sphenoethmoid recess superior and posterior to the superior concha. The posterior ethmoid air cells drain into the superior meatus. The nasofrontal duct,

How well did you know this?
1
Not at all
2
3
4
5
Perfectly
15
Q

The mylohyoid muscle is innervated via which of the following nerves?

A ) Accessory

B ) Glossopharyngeal

C ) Hypoglossal

D ) Mandibular

E ) Maxillary

A

The correct response is Option D.

The mylohyoid muscle is a flat, triangular muscle that with its opposite forms the muscular floor of the oral cavity. It originates from the oblique line of the mandible from the mandibular symphysis to the last molar tooth and inserts along the body of the hyoid bone in median fibrous raphe from the mandibular symphysis to the hyoid bone. The mylohyoid muscle elevates the floor of the mouth during swallowing and elevates the hyoid bone, which pushes the tongue upward during swallowing or tongue protrusion. This muscle also lowers the mandible and assists in opening the mouth, mastication, sucking, and blowing.

The primary motor innervation to the mylohyoid muscle is the mylohyoid branch of the inferior alveolar nerve of cranial nerve V (mandibular nerve). This nerve is a branch of the mandibular or third division of the trigeminal nerve. The two major nerves of the third division of the trigeminal nerve are the lingual nerve and the inferior alveolar nerve. The maxillary nerve is a branch of the second division of the trigeminal nerve and provides sensation to the ipsilateral face, side of the nose, lip, and upper teeth. The hypoglossal nerve (cranial nerve XII) provides motor innervation to the tongue. This nerve supplies all intrinsic muscles and all but one extrinsic muscle of the tongue. The exception is the palatoglossus muscle, which is supplied by cranial nerve X. The spinal accessory nerve, which is cranial nerve XI, supplies motor innervation to the sternocleidomastoid and trapezius muscles. The glossopharyngeal nerve (cranial nerve IX) is a complex nerve that supplies efferent nerve innervation to the parotid gland and carries sensations to the carotid body and carotid sinus. This nerve also provides general sensation to the posterior one third of the tongue, the skin of the external ear, and the internal surface of the tympanic membrane and supplies taste for the posterior one third of the tongue.

How well did you know this?
1
Not at all
2
3
4
5
Perfectly
16
Q

The 4-month-old boy shown has congenital midline neck and mandibular anomalies, including a mild clefting of the mandible and atrophic skin in the central anterior neck. These anomalies are a result of the incomplete fusion of which of the following paired branchial arches?

A ) First

B ) Second

C ) Third

D ) Fourth

E ) Fifth

A

The correct response is Option B.

Congenital midline cervical clefts are a result of a failure of fusion of the paired second branchial arches in the midline during embryogenesis. It is a rare anomaly of the anterior neck diagnosed on the basis of the clinical presentation at birth. The most commonly reported features are a cleft of reddened, thinned tissue in the midline, a protuberance of skin superiorly, a blind epithelium-lined sinus tract caudally, and a fibrous subcutaneous cord. The cord may limit neck movement. There is some degree of retrognathia and variable degrees of clefting of the lip and mandible. Other anomalies that have been found in association with midline cervical cleft are thyroglossal duct cysts and ectopic bronchogenic cysts.

Branchial arches begin to develop in the fourth week as neural crest cells migrate into the head and neck region. By the end of the fourth week, four pairs of branchial arches are visible externally. The fifth and sixth arches are small and not yet visible on the surface of the embryo. The branchial arches are separated by prominent clefts called branchial grooves.

How well did you know this?
1
Not at all
2
3
4
5
Perfectly
17
Q

A 70-year-old man is scheduled to undergo wedge resection of a lesion on the lower lip. Physical examination of the lower lip shows an ulcerated, well-circumscribed 5-mm lesion 1 cm medial to the right oral commissure. Which of the following foramina is the most appropriate site for instillation of anesthetic for the procedure?

A ) Inferior alveolar

B ) Infraorbital

C ) Mental

D ) Nasopalatine

E ) Sphenopalatine

A

The correct response is Option C.

The most appropriate area for block infiltration is an intraoral blockade of the mental nerve. The inferior alveolar nerve travels through the mandible entering at the medial ramal border and exits at the mental nerve at the mental foramen at the level of the first and second bicuspid teeth, 1 cm below the alveolar ridge.

Blockage of the inferior alveolar nerve foramen requires injection at its entry in the posterior mandible, or within the bony canal €”an area not easily accessible.

Numbness of the upper lip and nasal sidewall can be accomplished with blockade of the infraorbital nerve.

Nasal palatine anesthetic produces numbness of the external nose.

Sphenopalatine nerve blockade produces anesthesia in the internal nose and palate.

How well did you know this?
1
Not at all
2
3
4
5
Perfectly
18
Q

The muscles of mastication are derived from which of the following branchial arches?
A ) First

B ) Second

C ) Third

D ) Fourth

E ) Fifth

A

The correct response is Option A.

The muscles of mastication (temporalis, masseter, and medial and lateral pterygoids), the mylohyoid, and anterior belly of the digastric, and the tensors veli palatini and tympani are derived from the first branchial arch.

Arch Nerve

First- Mandibular Trigeminal, V2, V3

Second- Hyoid Facial (VII)

Third Glossopharyngeal (IX)

Fourth and Sixth Superior laryngeal branch of vagus (X) Recurrent laryngeal branch of vagus (X)

Muscles

Muscles of mastication Mylohyoid and anterior belly of digastric Tensor tympani Tensor veli palatini

Muscles of facial expression Stapedius Stylohyoid Posterior belly of digastric

Stylopharyngeus

Cricothyroid Levator veli palatini Constrictors of pharynx Intrinsic muscles of

larynx

How well did you know this?
1
Not at all
2
3
4
5
Perfectly
19
Q

A 48-year old woman comes to the office because she has had persistent epiphora since she sustained a naso-orbital-ethmoid fracture one year ago. Jones I fluorescein dye test is performed. If the results of this test are normal, in which of the following locations is the dye most likely to be found?

(A) Inferior meatus

(B) Lower canaliculus

(C) Middle meatus

(D) Nasolacrimal duct

(E) Upper canaliculus

A

The correct response is Option A.

Epiphora is the accumulation of tears that are not evacuated by the lacrimal drainage system. The tears will often overflow onto the cheek. Injuries to the lacrimal drainage system have been reported to occur in 5% to 21% of patients sustaining naso €‘orbito €‘ethmoid trauma. Jones I and Jones II dye testing can be used to diagnose the level of obstruction.

In a Jones I dye test, fluorescein dye is instilled into the conjunctival sac. A cotton €‘tipped applicator is placed under the inferior turbinate, the site of the inferior meatus. In a positive test, dye flows through the lacrimal system and exits at the inferior meatus. Dye is recovered on the applicator. In a negative test, no dye is recovered. This indicates an obstruction in the lacrimal drainage system.

The Jones II dye test is performed to localize the level of obstruction within the drainage system. This test is performed immediately after the Jones I test. An irrigation cannula is inserted into the punctum. Saline is irrigated through the cannula. If dye-stained fluid is obtained in the nose, the test is positive, and a partial obstruction of the lacrimal drainage system is diagnosed. If no dye is obtained in the nose, the test is negative. If dye exits the other canaliculus, the obstruction is in the lower portion of the system. If no dye returns through the other canaliculus, but dye refluxes through the same canaliculus, then the obstruction is in the upper part of the system.

How well did you know this?
1
Not at all
2
3
4
5
Perfectly
20
Q

An otherwise healthy 20-year-old woman comes to the office because she has had a painless lump on the left floor of the mouth for the past three weeks. The lump has increased in size during the past week. Physical examination shows a fluctuant, 2-cm blue mass that is not tender to palpation. Which of the following is the most likely diagnosis?

(A) Lingual thyroid

(B) Ranula

(C) Thyroglossal duct cyst

(D) Torus

(E) Vascular anomaly

A

The correct response is Option B.

A ranula is a mucocele or mucous extravasation phenomenon in the floor of the mouth, arising from the ducts of the sublingual or submandibular glands, often as a sequela of obstruction of the sublingual gland. It usually presents as a unilateral swelling of the floor of the mouth that is fluctuant and tinted blue or glossy white. Treatment includes marsupialization or surgical excision including the sublingual gland. The ranula may herniate through the muscles of the floor of the mouth and present as a €œplunging ranula € or cervical mass.

A lingual thyroid is an uncommon condition in which the embryonic thyroid gland does not descend into the neck and presents as a firm, nontender mass at the tongue base.

A thyroglossal duct cyst represents residual epithelium-lined tracts that trace the path of descent of the thyroid and can be found anywhere in the paramedial region of the neck but are usually in the midline.

A torus is an exostosis and is a slow-growing, hard mass arising from the palate or mandible. The enlargement consists of bone covered by mucosa. Excision is indicated for tori that become symptomatic or bothersome.

Vascular anomalies are present at birth and grow with the patient. It would be unusual for a vascular anomaly to initially present in an adult.

How well did you know this?
1
Not at all
2
3
4
5
Perfectly
21
Q

At which of the following intraosseous locations is the inferior alveolar nerve farthest from the buccal cortex?

(A) Angle

(B) First molar

(C) Ramus

(D) Second premolar

(E) Third molar

A

The correct response is Option B.

The inferior alveolar nerve (IAN) is closest to the lingual cortex at the level of the first and second molars. The anatomy of the IAN is relevant in orthognathic and trauma surgery. The incidence of nerve transection during sagittal split osteotomy is 3.5% and usually occurs at the level of the third molar. Osteotomy design and chisel placement are based on knowledge of the anatomy of the IAN. The IAN enters the mandible on the medial side of the ramus approximately 10 mm below the sigmoid notch. It then courses through the canal that is closest to the buccal cortical plate in the region of the ramus and angle and down to the third molar; the average distance is 1.8 ±1 mm. The nerve then swerves away at a position of 4.1 ±1 mm from the buccal cortex as it passes the region of the first and second molars. As it traverses the mandibular body, it is lowest and closest to the inferior cortex (7.5 ±1.5 mm) near its exit site at the level of the first molar and second premolar.

How well did you know this?
1
Not at all
2
3
4
5
Perfectly
22
Q

In the developing embryo, the stapes is formed from tissues of which of the following visceral arches?

(A) First

(B) Second

(C) Third

(D) Fourth

(E) Fifth

A

The correct response is Option B.

The first visceral arch ultimately forms the malleus, incus and the structures of the mandible. The second arch forms the stapes, the styloid and the facial musculature. The third arch forms the stylopharyngeus. The fourth, fifth and sixth arches contribute to the pharyngeal and laryngeal muscles as well as the thyroid cartilage.

How well did you know this?
1
Not at all
2
3
4
5
Perfectly
23
Q

A 45 year old man has persistent maxillary sinusitis subsequent to open reduction and internal fixation of pan €‘facial fractures and a failed course of antibiotics. Surgical enlargement of the ostium of the sinus is planned as a drainage procedure via an endoscopic approach. Direction of the endoscope into which of the following areas within the nasal cavity is most appropriate?

(A) Inferior concha

(B) Inferior meatus

(C) Middle meatus

(D) Superior concha

(E) Superior meatus

A

C Middle Meatus

How well did you know this?
1
Not at all
2
3
4
5
Perfectly
24
Q

Dermoid cysts of the nasal dorsum, anterior encephaloceles, and nasal gliomas are all thought to result from failed closure of which of the following structures?

(A) Anterior fontanelle

(B) Dorsum sellae

(C) Fonticulus frontalis

(D) Foramen rotundum

(E) Metopic suture

A

(C) Fonticulus frontalis

How well did you know this?
1
Not at all
2
3
4
5
Perfectly
25
Q

Which of the following foramina of the middle cranial fossa hosts passage of the oculomotor (III), trochlear (IV), and abducens (VI) nerves?

(A) Optic canal

(B) Ovale

(C) Rotundum

(D) Spinosum

(E) Superior orbital fissure

A

The correct response is Option E.

The foramen that allows passage of the oculomotor (III), trochlear (IV), and abducens (VI) cranial nerves is the superior orbital fissure, which is located at the orbital apex. This foramen also transmits the lacrimal, frontal, and nasociliary nerves; the sympathetic branches; the superior and inferior ophthalmic veins; the orbital branch of the middle meningeal artery; and the recurrent branch of the lacrimal artery. Hemorrhage or traumatic edema inside this foramen causes a unique collection of symptoms due to compression and loss of function of the traveling nerves, termed superior orbital fissure syndrome. The apex of the orbit also contains two other foramina: the optic canal, which transmits the optic (II) nerve and the ophthalmic artery, and the inferior orbital fissure, which allows passage of the infraorbital and zygomatic nerves. The foramen rotundum communicates between the middle cranial fossa and the pterygopalatine fossa. The maxillary division of the trigeminal (V) nerve is the only structure that passes through this foramen. The foramen ovale links the middle cranial fossa to the infratemporal fossa and transmits the mandibular division of the trigeminal nerve, the lesser petrosal branch of the glossopharyngeal (IX) nerve, the accessory meningeal branch of the maxillary artery, and an emissary vein. The foramen spinosum, located adjacent to the foramen ovale, also sits between the middle cranial and the infratemporal fossae. The foramen spinosum contains the meningeal branch of the mandibular division of the trigeminal nerve and the middle meningeal artery and vein.

How well did you know this?
1
Not at all
2
3
4
5
Perfectly
26
Q

Which of the following nerves supplies sensation to the superior anterior aspect of the helix of the ear?

(A) Arnold

(B) Auriculotemporal

(C) Great auricular

(D) Greater occipital

(E) Lesser occipital

A

The correct response is Option B.

Sensation to the external ear is provided by the great auricular nerve, auricular branch of cranial (X) nerve (Arnold nerve), the auriculotemporal nerve, a branch of the mandibular branch of cranial (V) nerve (trigeminal nerve), and the lesser occipital nerve. Sensation to the superior anterior aspect of the helix is provided by the auriculotemporal nerve. The Arnold nerve provides sensation to the ear canal, whereas the posterior aspect of the external ear is provided by the lesser occipital nerve and most of the inferior two thirds of the external ear is provided by the great auricular nerve. The greater occipital nerve provides sensation to the skin and the posterior aspect of the scalp.

How well did you know this?
1
Not at all
2
3
4
5
Perfectly
27
Q

An otherwise healthy 15-year-old boy is brought to the office by his parents because he has had painless swelling of the hard palate for the past year. A photograph of his mouth is shown. Physical examination shows a lesion that is solid, firm, fixed, noncompressible, nontender, and covered with healthy palatal mucosa. Which of the following is the most likely diagnosis?

(A) Craniofacial fibrous dysplasia

(B) Maxillary torus

(C) Skull base glioma

(D) Squamous cell carcinoma

(E) Submucous cleft palate

A

The correct response is Option B.

In an otherwise healthy young patient with a solid asymptomatic lesion consistent with bone on the hard palate, the most likely diagnosis is a maxillary torus or torus palatinus.

Maxillary and mandibular tori are benign osteoblastic tumors and tend to be well encapsulated, circumspect, submucosal, and expand gradually without invading adjacent structures. Torus palatinus occurs on the midline of the hard palate and requires treatment only if it becomes symptomatic or interferes with function.

Fibrous dysplasia is a benign tumor of bony origin that may present as a component of McCune-Albright syndrome (polyostotic fibrous dysplasia) or as a localized condition of the craniofacial skeleton, usually the upper facial skeleton. An isolated palatal occurrence of fibrous dysplasia would be highly unlikely.

Gliomas are believed to be encephaloceles that have lost their intracranial connections. External gliomas usually appear at or just lateral to the nasal root as reddish, firm, noncompressible, lobular lesions with cutaneous telangiectasia. The do not transilluminate or pulsate. Skull base gliomas are usually intranasal, and while potentially being associated with bony defects, would not present in this fashion.

Although squamous cell carcinoma frequently presents as an intraoral mass, it is not the most likely diagnosis in a healthy, young patient with a hard bony lesion of the palate.

Submucous cleft palates present with bifid uvulas, a bony notch or defect at the edge of the hard palate, and a zona pellucida, or thin, bluish, strip of mucosa in the midline of the soft palate secondary to diastasis of the levator muscle.

How well did you know this?
1
Not at all
2
3
4
5
Perfectly
28
Q

A 37-year-old man is brought to the emergency department one hour after he sustained injuries to the face in a motor vehicle collision. Physical examination shows lacerations around the bridge of the nose and mobility of the bony pyramid. Radiographs show a fracture of the nasal bones. Absence of sensation of the nasal septum and lateral nasal wall is noted. This patient has most likely sustained injuries to which of the following nerves?

(A) Anterior ethmoidal

(B) Frontal

(C) Infraorbital

(D) Infratrochlear

(E) Lacrimal

A

The correct response is Option A.

The ophthalmic branch or first division of the trigeminal nerve enters the orbit via the superior orbital fissure. The three sensory branches include the lacrimal, frontal, and nasociliary nerves. The nasociliary nerve courses above the optic nerve and below the superior rectus muscle. Its first branch is the posterior ethmoidal nerve, which provides sensation to the posterior ethmoid sinuses. The terminal branches of the nasociliary nerve are the anterior ethmoidal nerve and the infratrochlear nerve. The former exits via a foramen of the same name, and ultimately reaches the roof of the nose, providing sensation to the septum and lateral wall. Its terminal branch, the dorsal nasal nerve, supplies innervation to the tip.

How well did you know this?
1
Not at all
2
3
4
5
Perfectly
29
Q

A 45 year old man who is employed as a construction worker comes to the office because of pain in the small and ring fingers of the nondominant left hand. He says the pain worsens and the fingers become blotchy on exposure to cold. He has smoked one pack of cigarettes daily for 30 years. Examination shows subungual hemorrhages in the small finger and a digital brachial index of 0.4. Arteriography shows segmental occlusion of the ulnar artery at the wrist. Which of the following is the most appropriate management of this patient €™s condition?

(A) Administration of a calcium channel blocker

(B) Intraarterial thrombolysis/fibrinolysis

(C) Resection and reconstruction of segmental ulnar artery

(D) Smoking cessation

(E) Stellate ganglion block

A

The correct response is Option C.

Hypothenar hammer syndrome (HHS) describes digital ischemic symptoms secondary to either occlusion or aneurysmal dilation of the ulnar artery adjacent to the hamate. Although typically related to activities that involve repetitive trauma (eg, use of vibrating tools) to the palm, evidence exists to suggest that the condition arises in vessels with preexisting abnormalities, even in the absence of trauma.

In cases of critical digital ischemia with characteristic segmental occlusion on arteriography, optimal treatment involves resection of the diseased segment and reconstruction. Reversed vein graft represents the standard bypass conduit; however, as in coronary artery bypass grafting, arterial conduits (inferior epigastric artery) have been suggested as better matched in size and less prone to aneurysmal dilation later.

Calcium channel blockers and sympathetic blockade may alleviate vasospastic responses in the patent vessels distal to the occlusion. These interventions may be combined with surgical therapy and can be considered as primary interventions in patients with less symptomatic hands and a digital brachial index less than 0.7.

Intraarterial thrombolysis should not be used in the setting of the chronic, occlusive disease of HHS.

Smoking has been associated with HHS, but cessation alone in the setting of severe ischemia will not reverse the changes.

How well did you know this?
1
Not at all
2
3
4
5
Perfectly
30
Q

A 7-year-old boy is brought to the office for consultation regarding the congenital nerve palsy shown. The affected nerve is derived from which of the following branchial arches?

(A) First

(B) Second

(C) Third

(D) Fourth

(E) Fifth

A

The correct response is Option B.

The facial nerve (cranial nerve VII) is derived from the second branchial arch and consists primarily of motor fibers that are distributed to the muscles of facial expression. Branchial arches begin to develop in the fourth week as neural crest cells migrate into the head and neck region. By the end of the fourth week, four pairs of branchial arches are visible externally. The fifth and sixth arches are small and not visible on the surface of the embryo yet. The branchial arches are separated by prominent clefts called branchial grooves.

How well did you know this?
1
Not at all
2
3
4
5
Perfectly
31
Q

A 76-year-old man undergoes a radial forearm flap procedure for reconstruction of a defect of the floor of the mouth resulting from excision of carcinoma. During dissection of vessels in the neck for microsurgical anastomosis, the surgeon has difficulty visualizing an appropriate recipient artery because of high bifurcation of the carotid artery. Which of the following muscles can be cut to provide better exposure of the recipient artery?

(A) Digastric

(B) Geniohyoid

(C) Omohyoid

(D) Sternohyoid

(E) Styloglossus

A

The correct response is Option A.

There are only two muscles that would be in the area of a high bifurcation of the carotid that may need to be cut for better exposure: posterior belly of the digastric and the stylohyoid, which span across the skull base to the hyoid. Because the stylohyoid is not mentioned, the digastric muscle is the correct choice. The microsurgeon usually divides the digastric muscle at the level of the fibrous loop for the intermediate digastric tendon. The other muscles are never dissected or divided during this procedure.

How well did you know this?
1
Not at all
2
3
4
5
Perfectly
32
Q

Which of the following muscles of facial expression is innervated on its anterior surface?

(A) Depressor anguli oris

(B) Levator anguli oris

(C) Levator labii superioris

(D) Orbicularis oris

(E) Zygomaticus major

A

The correct response is Option B.

The muscles of facial expression receive their innervation from cranial nerve VII (facial nerve). The facial nerve innervates the muscles of facial expression along the posterior surface of the muscle in most cases. The exceptions to this rule include the levator anguli oris, the buccinator, and the mentalis muscle. At the modiolus, the fibers of the levator anguli oris coalesce with fibers of the zygomaticus major, orbicularis oris, risorius, buccinator, and depressor anguli oris. The levator anguli oris lies deep to the other muscles and receives its innervation along its anterior surface.

How well did you know this?
1
Not at all
2
3
4
5
Perfectly
33
Q

Formation of the primary palate begins during which of the following weeks of gestation?

(A) First

(B) Third

(C) Fifth

(D) Eighth

(E) Eleventh

A

The correct response is Option C.

Formation of the primary palate begins during the fifth week of gestation and is completed by the end of the sixth week of gestation. Anatomically, it is located anterior to the incisive foramen. The premaxilla only contains the central and lateral incisors.

How well did you know this?
1
Not at all
2
3
4
5
Perfectly
34
Q

For each structure of the neck, choose the fascial plane in which it is enveloped (A -C).

(A) Pretracheal fascia

(B) Prevertebral fascia

(C) Superficial fascia

142.

Trapezius muscle

143.

Anterior scalene muscle

A

The correct response for Item 142 is Option C and for Item 143 is Option B.

The superficial fascia envelops the sternocleidomastoid, the trapezius, and the suprahyoid muscles. The prevertebral fascia incorporates all of the scalene and paravertebral muscles. The pretracheal fascia incorporates the thyroid and trachea.

How well did you know this?
1
Not at all
2
3
4
5
Perfectly
35
Q

Which of the following muscles of the mouth is innervated by the mandibular branch of the facial (VII) nerve?

(A) Buccinator

(B) Depressor anguli oris

(C) Levator anguli oris

(D) Orbicularis oris

(E) Risorius

A

The correct response is Option B.

The other mimetic muscles innervated by the mandibular branch of the facial nerve include the depressor labii inferioris and the mentalis muscle. These muscles control movement of the lower lip to either pull it down and laterally or to protrude it. Paralysis of the nerve results in elevation of the angle of the mouth and of the lower lip on the affected side. This nerve is at risk for injury during rhytidectomy or neck lift procedures, parotidectomy, and neck dissection.

The buccinator, levator anguli oris, orbicularis oris, and risorius are all innervated by the buccal branch of the facial nerve. Anatomic variation may be encountered with the risorius muscle, which is rarely innervated by the mandibular branch.

How well did you know this?
1
Not at all
2
3
4
5
Perfectly
36
Q

Which of the following structures passes through the pterygomaxillary fissure?

(A) Mandibular artery
(B) Mandibular nerve
(C) Maxillary artery
(D) Maxillary nerve

A

The correct response is Option C.

The pterygomaxillary fissure appears on lateral cephalograms as an upside-down teardrop. It serves as a radiographic point of orientation. The posterior border of this opening is the anterior aspect of the pterygoid plates. The anterior border is the posterior aspect of the maxilla, and the superior border is the pterygopalatine fossa and sphenoid bone and the orbital process of the palatine bone. The inferior orbital fissure and the pterygomaxillary fissure are oriented at right angles to each other. They are separated by the small pterygopalatine fossa. The pterygomaxillary fissure connects the small pterygopalatine fossa with the much larger infratemporal fossa. It transmits the terminal branches of the maxillary artery and veins. The inferior orbital fissure transmits the maxillary nerve and branches of the pterygopalatine ganglion from the pterygopalatine fossa to the orbit. The mandibular nerve and branches pass through the infratemporal fossa but not through the pterygomaxillary fissure. Appropriate care must be taken during separation of the face from the pterygoid plates to avoid injury to the maxillary artery and veins. After separation of these components, down-fracture is often performed using digital pressure alone.

How well did you know this?
1
Not at all
2
3
4
5
Perfectly
37
Q

A 56-year-old woman undergoes reconstruction of the mandible using a free fibular flap. During the procedure, the microsurgeon dissects the external carotid artery; however, the superior thyroid artery cannot be located for anastomosis. Which of the following branches of the external carotid artery is the next distal vessel that would be suitable for anastomosis?
(A) Lingual
(B) Occipital
(C) Posterior auricular
(D) Maxillary

A

The correct response is Option A.

During the dissection of the neck vessels, the microsurgeon must be comfortable with the anatomy of the entire neck including the orientation and suitability of the available arteries. The branches of the external carotid artery, in order from proximal to distal, are as follows: superior thyroid, ascending pharyngeal, lingual, occipital, facial, posterior auricular, and maxillary arteries.

38
Q

A 64-year-old man comes to the office because he has numbness of the left medial cheek and infraorbital area. Physical examination shows a mass of the hard palate. Incisional biopsy shows adenoid cystic carcinoma. Further evaluation is most likely to show tumor involvement of which of the following foramina at the base of the skull?
(A) Anterior ethmoid foramen
(B) Foramen ovale
(C) Foramen rotundum
(D) Jugular foramen
(E) Stylomastoid foramen

A

The correct response is Option C.

Numbness in the left medial cheek and left infraorbital area suggest that the tumor has invaded the infraorbital nerve, which exits the middle cranial fossa from the foramen rotundum. Approximately 80% of patients with adenoid cystic carcinoma have perineural spread. The anterior ethmoid foramen emerges from the frontoethmoid suture line and contains the anterior ethmoid vessels. The foramen ovale and the stylomastoid foramen, respectively, contain the mandibular nerve (V3) and the facial nerve (cranial nerve VII). Cranial nerves IX (glossopharyngeal), X (vagus), and XI (spinal accessory) emerge from the jugular foramen.

39
Q

Which of the following muscles elevates the mandible?
(A) Digastric
(B) Genioglossus
(C) Geniohyoid
(D) Medial pterygoid
(E) Mylohyoid

A

The correct response is Option D.

The mandible is subject to muscular forces, which tend to add to the instability of certain fractures and necessitate treatment. The masseter, temporalis, and medial pterygoid muscles elevate the mandible, and the geniohyoid, genioglossus, mylohyoid, and digastric muscles tend to depress the mandible. The lateral pterygoid muscle inserts into the capsule of the temporomandibular joint and tends to remain attached and pull the condyle head medial when there is a high fracture.

40
Q

Which of the following muscles can function to close off the oral cavity from the oropharynx?
(A) Levator veli palatini
(B) Musculus uvulae
(C) Palatoglossus
(D) Palatopharyngeus
(E) Tensor veli palatini

A

The correct response is Option C.

The palatoglossus is a paired muscle that elevates the posterior tongue and pulls it against the soft palate, separating the oral cavity from the oropharynx. The muscle attaches to the side of the tongue on one end and the palatine aponeurosis on the other.

The levator veli palatini elevates the soft palate during swallowing and yawning. It attaches superiorly to the cartilage of the auditory tube and the petrous part of the temporal bone and inferiorly to the palatine aponeurosis.

The palatopharyngeus muscle is part of the palatopharyngeal arch, attaching to the lateral wall of the pharynx, the hard palate, and the palatine aponeurosis. During swallowing, it tenses the soft palate while pulling the walls of the pharynx superiorly, medially, and anteriorly, effectively closing off the nasopharynx from the oropharynx.

The tensor veli palatini also tenses the soft palate and opens the eustachian tube during yawning and swallowing. It extends from the palatine aponeurosis to the medial pterygoid plate and spine of the sphenoid bone and the cartilage of the eustachian tube.

The musculus uvulae pulls the uvula superiorly and shortens it. The muscle fills the substance of the uvula, attaching to its mucosa and to the posterior nasal spine and palatine aponeurosis.

41
Q

A 6-year-old boy who underwent repair of a unilateral cleft lip and palate in infancy is brought to the office by his parents because he has had a sensation of numbness in the left anterior mobile tongue for the past two days, since he underwent repair of a palatal fistula during orotracheal intubation. During that procedure, the oral cavity and palate were retracted for two hours with a Dingman retractor. Which of the following nerves was most likely damaged during the procedure?
(A) Glossopharyngeal (IX)
(B) Lingual
(C) Mental
(D) Recurrent laryngeal
(E) Superior laryngeal

A

The correct response is Option B.

The lingual nerve provides sensation to the anterior two thirds of the tongue. Injury to the lingual nerve is a rare complication of airway manipulation. Virtually all equipment associated with airway management and/or oral cavity retraction has been implicated in its damage. Examples include direct laryngoscopy, tracheal intubation, laryngeal mask airway, and tonsillectomy. The etiology of lingual nerve damage is likely compression of the nerve in the floor of mouth against the mandible caused by forceful compression of the floor of mouth. The mental nerve exits the mental foramen just below the first or second premolar and supplies sensation to the lower lip and chin. The superior laryngeal nerve provides sensory innervation to the supraglottis. The glossopharyngeal nerve provides sensory innervation and taste sensation to the base of the tongue. The recurrent laryngeal nerve provides motor innervation to the laryngeal musculature.

42
Q

Which of the following best describes the pathway of the accessory (XI) nerve after it enters the posterior triangle of the neck?
(A) Within the subcutaneous fat
(B) Within the carotid sheath
(C) Within the inferior portion of the sternocleidomastoid muscle
(D) Deep to the investing fascia
(E) Deep to the prevertebral fascia

A

The correct response is Option D.

The accessory nerve descends through the jugular foramen into the neck between the internal carotid and internal jugular vein. It crosses the internal jugular vein and is joined by fibers from the ventral ramus of C2. It gives off a motor branch to the sternocleidomastoid muscle before passing deep to or through this muscle. It emerges on the posterior border of the sternocleidomastoid muscle always in the upper half of this muscle. It enters the posterior triangle of the neck passing obliquely across the floor over the levator scapulae just deep to the investing fascia and superficial to the prevertebral fascia. It supplies motor fibers to the trapezius.

43
Q

The papilla of the parotid duct is most commonly located adjacent to which of the following maxillary teeth?
(A) First bicuspid
(B) Second bicuspid
(C) First molar
(D) Second molar
(E) Third molar

A

The correct response is Option D.

The papilla of the parotid is located most frequently adjacent to the maxillary second molar as it empties the secretions of the parotid gland via Stensen’s duct.

44
Q

Which of the following structures travels through the foramen rotundum?
(A) First division of the trigeminal (V) nerve
(B) Second division of the trigeminal (V) nerve
(C) Third division of the trigeminal (V) nerve
(D) Optic (II) nerve
(E) Middle meningeal artery

A

The correct response is Option B.

The maxillary division of the trigeminal nerve travels through the foramen rotundum. The ophthalmic division of the trigeminal nerve travels through the superior orbital fissure. The mandibular division of the trigeminal nerve travels through the foramen ovale. The foramen spinosum carries the middle meningeal artery. The optic nerve travels through the optic foramen.

45
Q

Which of the following muscles is included in a facial artery musculomucosal flap?

(A) Buccinator
(B) Depressor anguli oris
(C) Levator labii superioris
(D) Orbicularis oris
(E) Zygomaticus major

A

The correct response is Option A.

Because the buccinator muscle is sandwiched between the facial artery and the oral mucosa, it must be included in a facial artery musculomucosal flap. The buccinator muscle originates from the pterygomandibular raphe and inserts into the orbicularis oris muscle and mucosa of the lateral lip elements.
The other muscles listed are supplied by the facial nerve but are not included in a facial artery musculomucosal flap because of their location. The depressor anguli oris and zygomaticus major muscles lie superficial to the plane of dissection for this flap. The levator labii superioris muscle inserts into the lateral half of the lip, but its origin is from the infraorbital margin of the maxilla. The orbicularis oris muscle is anterior to the plane of dissection for the facial artery musculomucosal flap.

46
Q

Which of the following muscles of the palate is innervated predominantly by the trigeminal (V) nerve?

(A) Levator veli palatini
(B) Musculus uvula
(C) Palatoglossus
(D) Palatopharyngeus
(E) Tensor veli palatini

A

The correct response is Option E.

The tensor veli palatini muscle is innervated predominantly by the mandibular branch of the trigeminal nerve (V3). In contrast, the other palatal muscles, including the levator veli palatini, musculus uvula, palatoglossus, and palatopharyngeus, are innervated predominantly by branches of the pharyngeal plexus of the vagus (X) nerve.

47
Q

A 65-year-old man undergoes operative removal of a basal cell carcinoma at the junction of the upper cheek and temporal region, followed by coverage of the resultant 10 _ 5-cm defect with a submental myocutaneous flap. This flap derives its blood supply from a branch of which of the following arteries?

(A) Facial
(B) Inferior thyroid
(C) Lingual
(D) Superior thyroid
(E) Transverse cervical

A

The correct response is Option A.

The submental flap is a myocutaneous flap that is useful in head and neck reconstruction. This flap provides a contour, color, and tissue texture that is suitably matched to the face. The flap is elevated below the level of the platysma muscle and includes the submental artery and vein, which are direct branches of the facial artery and vein. The flap can be transposed to cover defects in the lower and central thirds of the face and into the inferior aspect of the upper third of the face.
The submental artery is a consistent branch of the facial artery and gives off one or two cutaneous perforators to the submental skin. The submental artery runs in relation to the anterior belly of the diagastric muscle. Of the choices listed, the facial artery is the most superior branch of the external carotid artery. The lingual artery provides the blood supply to the tongue, the superior thyroid and inferior thyroid arteries provide the blood supply to the thyroid gland, and the transverse cervical artery gives off a descending branch, which provides the blood supply to the trapezius muscle.

48
Q

The parotid (Stensen) duct opens into the mouth at which of the following levels?

(A) Lower first molar
(B) Lower second molar
(C) Upper first molar
(D) Upper second molar
(E) Upper third molar

A

The correct response is Option D.

The parotid (Stensen) duct opens into the mouth at the level of the upper second molar. It is about 4 to 7 cm long, extends from the anterior border of the superficial lobe of the parotid gland, and is frequently accompanied by accessory parotid tissue. The duct runs anteriorly over the masseter muscle about halfway between the zygomatic arch and the angle of the mouth. It turns medially beyond the anterior border of the masseter muscle, pierces the buccinator muscle, and opens into the mouth through the parotid papilla at about the level of the upper second molar. The zygomatic and buccal branches of the facial (VII) nerve may cross superficially over the parotid duct.

The location of the parotid duct has implications for placement of surgical incisions. Vertical incisions at the anterior border of the parotid gland may cause injury to the parotid duct or branches of the facial (VII) nerve. Horizontal incisions are less likely to damage these structures. Thus, parotid abscesses should be drained through a horizontal incision. Because the parotid gland is divided by fascial septa that extend from its capsule, a parotid abscess may be compartmentalized. Therefore, multiple horizontal incisions may be needed for adequate drainage. Also, a parotid abscess may not be fluctuant because of the septa. Pitting edema over the parotid gland usually indicates an abscess.

49
Q

Which of the following muscles contributes to Passavant’s ridge?

(A) Middle pharyngeal constrictor
(B) Styloglossus
(C) Stylopharyngeus
(D) Superior pharyngeal constrictor
(E) Tensor veli palatini

A

The correct response is Option D.

During gagging, forceful contraction of the superior pharyngeal constrictor and levator palatini muscles may produce Passavant’s ridge, which is a bulge on the posterior pharynx above the arch of the atlas. In patients with velopharyngeal incompetence (VPI), this ridge may be activated as a compensatory mechanism and may serve as a reference point during surgery to correct VPI.

The other muscles listed are not involved in Passavant’s ridge. The middle pharyngeal constrictor muscle arises from the hyoid bone and is inferior to the superior pharyngeal constrictor. The styloglossus and stylopharyngeus muscles both originate from the medial aspect of the styloid process. Then the styloglossus passes down to the tongue; the stylopharyngeus passes down to the pharyngeal wall. The tensor veli palatini muscle originates broadly from the scaphoid fossa of the medial pterygoid plate and the lateral eustachian tube. The fibers of this muscle pass around the pterygoid hamulus and attach to the shelf of the posterior hard palate.

50
Q

Actions of the levator veli palatini muscle include retracting the soft palate and which of the following?

(A) Closing the eustachian tube
(B) Drawing the uvula anteriorly
(C) Narrowing the faucial isthmus
(D) Opening the eustachian tube
(E) Widening the faucial isthmus

A

The correct response is Option D.

The soft palate consists of the levator veli palatini muscle as well as the muscles of the uvula and the palatopharyngeus, palatoglossus, and tensor veli palatini muscles. The levator veli palatini muscle raises and retracts the soft palate to bring it in touch with the posterior pharyngeal wall. It also opens the eustachian tube. The uvular muscles draw the uvula superiorly and anteriorly. The palatopharyngeus and palatoglossus muscles lower the soft palate and narrow the faucial isthmus. The tensor veli palatini muscle tenses and lowers the soft palate and opens the eustachian tube.

In the repair of a cleft of the soft palate, the tensor veli palatini muscle is difficult to approximate because the tendon runs over the hamulus. In contrast, the levator veli palatini muscle can be easily approximated. To avoid separation of the tensor tendon, it can be dissected off the hamulus by fracturing the hamulus. Paralysis of the soft palate may cause nasal regurgitation and interfere with speech. In unilateral paralysis of the soft palate, the uvula deviates toward the normal side during elevation.

51
Q

The foramen rotundum is situated within which of the following structures?

(A) Anterior cranial fossa
(B) Cribriform plate
(C) Lesser petrosal sinus
(D) Orbit
(E) Sphenoid bone

A

The correct response is Option E.

The foramen rotundum is situated within the sphenoid bone in the middle cranial fossa, not on a lesser wing of the sphenoid bone, which is located in the anterior cranial fossa. The foramen rotundum transmits the maxillary branch (V2) of the trigeminal (V) nerve from the middle cranial fossa into the pterygopalatine fossa.

The anterior cranial fossa contains the frontal and ethmoid bones and the lesser wings of the sphenoid bone. The foramina of the anterior cranial fossa include the foramen cecum, foramina of the cribriform plate, and the anterior and posterior ethmoidal canals.

The cribriform plate (the horizontal component of the ethmoid bone) comes in contact with the anterior cranial fossa on one side and with the nasal cavity on the other. The cribriform plate contains multiple foramina for the olfactory (I) nerves.

The lesser petrosal sinus is located in the anterior wall of the petrous part of the temporal bone in the middle cranial fossa. It transmits the lesser petrosal nerve. The orbit has three important foramina: the optic canal and the superior orbital fissure, which connect posteriorly with the middle cranial fossa, and the inferior orbital fissure, which faces the pterygopalatine fossa posteriorly. The optic canal transmits the optic (II) nerve and the ophthalmic artery. The superior orbital fissure transmits the oculomotor (III), trochlear (IV), and abducens (VI) nerves and the ophthalmic division of the trigeminal (V) nerve. The inferior orbital fissure transmits the maxillary and zygomatic nerves and the infraorbital vessels.

52
Q

Which of the following muscles of facial animation is innervated via its superficial surface?

(A) Buccinator
(B) Depressor anguli oris
(C) Orbicularis oculi
(D) Orbicularis oris
(E) Zygomaticus major

A

The correct response is Option A.

Understanding of the muscles of facial animation and its innervation is crucial for safe facial aesthetic surgery. The layers of the face include the skin, subcutaneous tissue, SMAS, fascia, and facial (VII) nerve. Dissection within the subcutaneous plane is safe because this plane does not contain the facial (VII) nerve. The SMAS is a heterogenous structure, consisting of facial muscles, fibrous tissue, and adipose tissue. Cephalad, the SMAS continues as the temporoparietal fascia. Dissection superficial to the SMAS layer is safe because the muscles of facial animation are innervated from below, except for the buccinator, mentalis, and levator anguli oris muscles, which are innervated via their superficial surfaces.
The depressor anguli oris, orbicularis oculi, orbicularis oris, and zygomaticus major are all innervated from their deep surfaces. When dissecting below the SMAS plane, the zygomaticus major muscle serves as an important landmark. In this plane, dissection should proceed more superficially to preserve innervation of the zygomaticus major muscle.

53
Q

A neonate has a reddish 1.5-cm mass of the nasal root with overlying cutaneous telangiectasias. A photograph is shown above. On physical examination, the mass is firm, noncompressible, and nonpulsatile. It does not transilluminate or change with Valsalva maneuver. Which of the following is the most likely diagnosis?

(A) Dermoid cyst
(B) Encephalocele
(C) Glioma
(D) Hemangioma
(E) Lipoma

A

The correct response is Option C.

The findings in this neonate are consistent with a glioma. Nasal gliomas are thought to originate as encephaloceles but fail to maintain their intracranial connections. They may be external, internal, or a combination of both. External gliomas typically appear at or just lateral to the nasal root. They are reddish, firm, noncompressible, lobular lesions that exhibit telangiectasias of the overlying skin, but do not transilluminate or pulsate. Bony defects, intracranial connections, and cerebrospinal fluid leakage occur only rarely. Histologic evaluation shows astrocytic neuroglial cells and fibrous and vascular connective tissue that is covered with skin or nasal mucosa.

A nasal dermoid cyst arises from a dermoid sinus, which is a cutaneous inward passage lined with stratified squamous epithelium. These masses can also be external or internal. An external nasal dermoid is a firm, noncompressible, nonpulsatile lesion that does not transilluminate and may be lobulated. Although bony defects are infrequent, cerebrospinal fluid leakage and meningitis may occur. Nasal dermoid cysts are derived from ectoderm and mesoderm, lined with squamous epithelium, and contain specialized adnexal structures such as hair follicles, pilosebaceous glands, and smooth muscles.

Encephaloceles involve herniation of cranial tissue through a skull defect. They may be classified as meningoceles (containing meninges only), meningoencephaloceles (containing meninges and brain), or meningoencephalocystoceles (containing meninges, brain, and part of the ventricular system). External, or sincipital, encephaloceles are soft, bluish, compressible, pulsatile masses that are located at the nasal root and transilluminate. They typically enlarge with crying and Valsalva maneuver.

Hemangiomas are raised lesions that arise from a proliferation of endothelial cells. Most appear shortly after birth and involute spontaneously after a period of rapid growth. Discoloration of the overlying skin is often associated.
Lipomas are soft, skin colored, compressible lesions that do not have cutaneous telangiectasias and do not transilluminate or pulsate. They may appear at the nasal root, but are not predisposed to that location.

54
Q

A 25-year-old woman is scheduled to undergo lip augmentation using dermal allograft. Blockade of the infraorbital and mental nerves will be performed bilaterally. In this patient, direct infiltration of local anesthetic is most likely to be required at which of the following sites?

(A) Central upper vermillion
(B) Central lower vermillion
(C) Commissures
(D) Cupid’s bow
(E) Philtrum

A

The correct response is Option C.

Because blockade of the infraorbital and mental nerves alone does not ensure adequate anesthesia of the oral commissures, direct infiltration of local anesthetic into the commissures is likely to be necessary. Although a portion of the sensibility of this region is supplied by contributions from the infraorbital and mental nerves, the greatest contributor is the buccal nerve, which is derived from the mandibular branch of the trigeminal nerve (V3). The buccal nerve supplies sensation to a large area of skin of the cheek that lies just lateral to the commissure and overlaps the area of the modiolus and buccinator muscle, as well as to a portion of the buccal mucosa on the opposite side of the buccinator. Although the infraorbital nerve may reach the upper half of the commissure and extend to 1.5 cm laterally, the border of this region follows an upward curve away from the commissure.

Bilateral blockade of the infraorbital nerve in the midline provides complete anesthesia to all central components of the upper lip, including the vermillion, cupid’s bow, and philtrum. In addition, bilateral mental nerve blocks effectively anesthetize the central section of the lower lip because the regions supplied by these nerves meet in the midline.

55
Q

Which of the following nerves supplies sensory innervation to the buccal mucosa?

(A) Trigeminal (V) nerve
(B) Facial (VII) nerve
(C) Glossopharyngeal (IX) nerve
(D) Vagus (X) nerve
(E) Lingual nerve

A

The correct response is Option A.

The buccal branch of the trigeminal (V) nerve provides sensation to the buccal mucosa. It is important for the surgeon to know the anatomy of this nerve branch to plan and perform neurotized free flap reconstruction and reinnervation of the intraoral cavity.

The buccal branch of the facial (VII) nerve innervates the muscles surrounding the buccal mucosa.

The glossopharyngeal (IX) and vagus (X) nerves do not provide sensory innervation to the intraoral mucosa.

The lingual nerve provides sensation to a portion of the tongue.

56
Q

A 38-year-old woman sustains an injury to the auriculotemporal nerve during superficial parotidectomy for removal of a mixed tumor. Which of the following is the most likely postoperative finding?

(A) Numbness of the concha, helix, lobule, and temporal skin
(B) Numbness of the tragus, external auditory canal, and temporal skin
(C) Numbness of the entire pinna and paralysis of the temporalis muscle
(D) Paralysis of the auricularis anterior, superior, and posterior muscles
(E) Paralysis of the temporalis muscle

A

The correct response is Option B.

Injury to the auriculotemporal nerve is most likely to result in numbness of the tragus, external auditory meatus, tympanum, and temporal skin. The auriculotemporal nerve is a branch of the mandibular division of the trigeminal nerve (V3). The fascicles of this nerve divide soon after the nerve originates to allow for passage of the middle meningeal artery. The nerve then courses between the sphenomandibular ligament and the neck of the condyle, emerges from behind the temporomandibular joint, and travels toward the posterior surface of the upper portion of the parotid gland, where it may be vulnerable to injury during parotidectomy. It ascends with the superficial temporal vessels over the posterior zygomatic arch and divides into three branches, which provide cutaneous sensory innervation to the tragus, external acoustic meatus, tympanic membrane, and temporal region. Minor branches of the auriculotemporal nerve convey secretomotor fibers to the parotid gland and articular fibers to the temporomandibular joint.

The concha and antihelix receive sensory innervation from the auricular branch of the vagus (X) nerve. Sensory innervation to the helix and lobule is supplied by the great auricular nerve and lesser occipital nerve, which are derived from C2-3.

The auricularis anterior, superior, and posterior muscles receive motor innervation from the temporal and posterior auricular branches of the facial (V) nerve.

The temporalis muscle is innervated by the deep temporal nerves, which are derived from the anterior, or motor, branch of the mandibular division of the trigeminal nerve (V3).

57
Q

Which of the following structures passes though the foramen ovale?

(A) Accessory nerves
(B) Optic (II) nerve
(C) Ophthalmic division of the trigeminal nerve (V1)
(D) Maxillary division of the trigeminal nerve (V2)
(E) Mandibular division of the trigeminal nerve (V3)

A

The correct response is Option E.

The mandibular division of the trigeminal nerve (V3) passes through the foramen ovale. This foramen is located in the region of the superior orbital fissure, which contains the nerves to the extraocular muscles, sympathetic fibers, and vessels and is found within the middle cranial fossa.

The accessory nerves, glossopharyngeal (IX) nerve, and vagus (X) nerve pass through the foramen jugulare.

The optic foramen transmits the optic (II) nerve and ophthalmic artery.

The ophthalmic division of the trigeminal nerve (V1) passes through the superior orbital fissure.

The foramen rotundum transmits the maxillary division of the trigeminal nerve (V2).

58
Q

The external auditory meatus develops from which of the following embryologic structures?

(A) First branchial arch
(B) Second branchial arch
(C) Third branchial arch
(D) First branchial groove
(E) Second branchial groove

A

The correct response is Option D.

Development of the six branchial arches occurs within the walls of the anterior foregut during the fourth week of gestation, as neural crest cells migrate into the future head and neck region and alternating ridges and depressions develop. Each branchial arch is composed of endoderm, ectoderm, and mesoderm. During development, a series of clefts forms to create the branchial grooves externally and the pharyngeal pouches internally. The branchial grooves are lined with surface ectoderm and the pharyngeal pouches are lined with foregut endoderm.

During the sixth week of gestation, six hillocks appear on the first (mandibular) and second (hyoid) branchial arches, which give rise to the auricle. The first branchial arch gives rise to the anterior (first through third) hillocks, and the second branchial arch gives rise to the posterior (fourth through sixth) hillocks. By the end of the eighth week of gestation, the auricle assumes its characteristic shape following differential growth and fusion of the hillocks.

The external auditory meatus develops from the dorsal aspect of the first branchial groove, which is a cleft between the first and second branchial arches.

The second, third, and fourth branchial grooves are obliterated within the cervical sinus during the later stages of embryologic development. The cervical sinus develops as a result of caudal overgrowth of the second branchial arch.

59
Q

Which of the following cranial nerves provides innervation to the temporalis muscle?

(A) Ophthalmic division of the trigeminal nerve (V1)
(B) Maxillary division of the trigeminal nerve (V2)
(C) Mandibular division of the trigeminal nerve (V3)
(D) Abducens (VI) nerve
(E) Facial (VII) nerve

A

The correct response is Option C.

The temporalis muscle receives its innervation primarily from the branches of the mandibular division of the trigeminal nerve (V3), which then exits the skull via the foramen ovale. The motor branches of the buccal, masseteric, and mandibular nerves are derived from V3 and act to innervate the temporalis muscle. This muscle is a large, powerful muscle of mastication that originates along the temporal crest of the skull and inserts into the coronoid process of the mandible.

The ophthalmic division of the trigeminal nerve (V1) provides sensation to the forehead and anterior scalp; this nerve branch exits the skull through the supraorbital foramen.

The maxillary division of the trigeminal nerve (V2) provides sensation to the cheek and upper lip and to the upper teeth via the superior alveolar nerve. This nerve branch is transmitted through the infraorbital foramen.

The abducens (VI) nerve provides motor innervation into the lateral rectus muscle of the eye.

The facial (VII) nerve provides motor innervation to the muscles of facial expression.

60
Q

Which of the following neural structures does NOT pass through the superior orbital fissure?

(A) Optic (II) nerve
(B) Oculomotor (III) nerve
(C) Trochlear (IV) nerve
(D) Abducens (VI) nerve
(E) Sympathetic nerve fibers

A

The correct response is Option A.

The superior orbital fissure transmits the oculomotor (III), trochlear (IV), and abducens (VI) nerves and sympathetic nerve fibers from the cavernous plexus. In patients who sustain high-velocity fractures of the orbital roof, the fractures may extend to involve the structures of the superior orbital fissure, resulting in a condition known as superior orbital fissure syndrome. This syndrome manifests as loss of ocular motion resulting from paralysis of the motor nerves that pass through the superior orbital fissure, but does not affect vision.

The optic (II) nerve and ophthalmic artery pass through the optic foramen, which is separated from the superior orbital fissure by the lesser wing of the sphenoid bone. Orbital apex syndrome, which involves injury to the optic nerve resulting from extension of the fracture into the optic canal, is characterized by loss of vision.

61
Q

Which of the following nerves passes through the stylomastoid foramen?

(A) Facial (VII)
(B) Glossopharyngeal (IX)
(C) Great auricular
(D) Mandibular
(E) Posterior auricular

A

The correct response is Option A.

The stylomastoid foramen is located anterior to the mastoid process and posterior to the styloid root. The facial nerve and stylomastoid artery pass through the stylomastoid foramen as they exit the temporal bone; from this point, the facial (VII) nerve gives off branches to sites throughout the face. The main trunk of the facial nerve supplies innervation to the posterior belly of the digastric muscle as it leaves the foramen and enters the parotid gland.

The glossopharyngeal (IX), vagus (X), and spinal accessory (XI) nerves and the internal jugular vein pass through the foramen jugulare. The mandibular nerve and accessory meningeal artery pass through the foramen ovale, which lies anteromedial to the foramen spinosum; the lesser petrosal nerve may also be transmitted through the foramen ovale.

The great auricular nerve is derived from the nerve roots of C2-3 and not from cranial nerves. It travels around the posterior border of the sternocleidomastoid muscle to innervate the skin of the mastoid area.

The posterior auricular nerve branches off from the facial nerve just after it passes through the stylomastoid foramen and supplies innervation to the skin behind the ear.

62
Q

Anesthesia of the nasal side wall is best accomplished through infiltration of which of the following nerves?

(A) Buccal
(B) Dorsal nasal
(C) Infraorbital
(D) Infratrochlear
(E) Zygomaticofacial

A

The correct response is Option C.

The infraorbital nerve, which is the terminal branch of the maxillary division of the trigeminal nerve (V2), supplies sensation to the nasal side wall, ala, upper lip, lower eyelid, and medial aspect of the upper cheek. Anesthesia of this nerve is best accomplished by introducing the needle just lateral to the nasal ala and directing it to a point 0.5 cm below the central section of the infraorbital rim, directly into the foramen rotundum.

The buccal nerve is a branch of the mandibular division of the trigeminal nerve (V3). It arises from the surface of the buccinator muscle to supply sensation to the central cheek. Appropriate anesthesia is achieved by injecting the nerve at its origin on the mandibular nerve between the sigmoid notch of the mandible and lateral pterygoid plate.
The dorsal nasal nerve and infratrochlear nerve are branches of the nasociliary nerve, which is derived from the ophthalmic division of the trigeminal nerve (V1). The dorsal nasal nerve branches from the anterior ethmoidal nerve, then emerges at the distal end of the nasal bones to supply sensation to the skin of the dorsal nose from that point distally to the nasal tip. The infratrochlear nerve supplies sensation to the skin of the root of the nose and medial upper eyelid.

The zygomaticofacial nerve is a branch of the zygomatic nerve, which is derived from the maxillary division of the trigeminal nerve (V2). It emerges through one or two foramina just lateral to the infraorbital rim and supplies sensation to the skin over the zygoma and upper portion of the central cheek.

63
Q

Which of the following muscles acts to elevate the mandible?

(A) Anterior digastric
(B) Buccinator
(C) Lateral pterygoid
(D) Medial pterygoid
(E) Posterior digastric

A

The correct response is Option D.

Knowledge of the muscles of the mandibular region and their functions in the movement of the mandible and displacement of fracture segments is crucial to accurately diagnosing and effectively treating facial fractures.

Contraction of the medial pterygoid muscle elevates the mandible and pulls it medially. This muscle arises inferiorly, laterally, and posteriorly from the medial surface of the pterygoid plate and inserts on the medial ramus and mandibular angle. A patient who sustains a fracture medial to the mandibular angle has displacement of the mandibular ramus medially and cephalad because of the forces of this muscle.

The anterior belly of the digastric muscle originates from the inside lower border of the symphysis and attaches to the lateral corner of the hyoid bone, while the posterior belly of the digastric extends between the hyoid bone and mastoid notch of the temporal bone. The digastric muscle is part of the suprahyoid musculature, which in a patient with a mandibular fracture would pull the anterior mandibular fragments posteroinferiorly.

The buccinator muscle arises from the pterygomandibular raphae to insert into the orbicularis oris muscle and the mucosa and skin of the lips. This muscle of facial expression is innervated by the buccal branches of the facial nerve and acts to flatten the cheek against the teeth. Because it does not arise within the condylar region, it does not exert any forces on the condyle or mandible.
The lateral pterygoid muscle arises from the lateral pterygoid plate of the sphenoid bone and extends posteriorly and horizontally to insert on the neck of the condyle and into the anterior margin of the articular disk of the temporomandibular joint. As the condyle is moved forward, the lateral pterygoid acts to protrude the mandible, then slides down the articular eminence, opening the mouth. In patients with subcondylar fractures, the muscle pulls the condylar head in an anteromedial direction, whereas in patients with unilateral fractures the contralateral pterygoid continues to protrude, resulting in deviation of the mandible to the side of the fracture.

64
Q

Which of the following branchial arches gives rise to the stylopharyngeus muscle?

(A) First
(B) Second
(C) Third
(D) Fourth
(E) Fifth

A

The correct response is Option C.
The stylopharyngeus is the only muscle derived from the third branchial arch. This muscle, which receives its innervation from the glossopharyngeal (X) nerve, is part of the intrinsic musculature of the larynx.

The muscles of mastication, the anterior belly of the digastric, the mylohyoid, the tensor tympani, and the tensor veli palatini are derived from the first branchial arch. These muscles are innervated by the trigeminal (V) nerve. The muscles of facial expression, the posterior belly of the digastric, the stylohyoid, and the stapedius are derived from the second branchial arch and are innervated by the facial (VII) nerve. The fourth branchial arch gives rise to the constrictor muscles of the pharynx, the cricothyroid, the levator veli palatini, the palatopharyngeus, and the palatoglossus. The superior laryngeal branch of the vagus (X) nerve supplies their innervation. The fifth branchial arch is only present in rudimentary forms and is often combined with the sixth branchial arch during any embryologic investigation. The intrinsic muscles of the larynx, except for the cricothyroid and stylopharyngeus, as well as the striated muscles of the esophagus are derivatives. These muscles are innervated by the recurrent laryngeal branch of the vagus (X) nerve.

65
Q

The hamulus bone acts as a pulley for which of the following muscles?

(A) Levator veli palatini
(B) Palatoglossus
(C) Palatopharyngeus
(D) Superior pharyngeal constrictor
(E) Tensor veli palatini

A

The correct response is Option E.
The hamulus, which is an inferior extension of the medial pterygoid plate, acts as a pulley for the tensor veli palatini muscle. This muscle arises from the medial pterygoid plate and consolidates into a tendon that changes course as it wraps around the hamulus, and then extends medially to form an aponeurosis in the anterior soft palate. The tensor veli palatini also inserts on the posterior hard palate.

The levator veli palatini muscle arises from the inferior surface at the apex of the petrous portion of the temporal bone, and then passes obliquely downward and medially to join fibers from the contralateral muscle in forming the palatine velum. This muscle primarily acts to move the palate. A fibrous linkage exists between the fibers of the levator veli palatini and the tensor veli palatini; therefore, a patient who sustains a fracture of the pterygoid hamulus may have increased medial displacement of these muscles.

The palatoglossus muscle arises from the soft palate and courses downward within the anterior tonsillar pillar to insert into the tongue.

The palatopharyngeus muscle arises from the soft palate, coursing down the posterior tonsillar arch to join the stylopharyngeus, which then inserts on the thyroid cartilage. This muscle can be used for flap transfer in a patient undergoing sphincter pharyngoplasty for pharyngeal reconstruction.

The superior pharyngeal constrictor muscle arises from the posterior aspect of the medial pterygoid plate and from the hamulus, pterygomandibular raphe, and alveolar process of the mandible. The fibers of this muscle wrap around the pharynx and insert on the median raphe to form Passavant’s ridge, which contributes to velopharyngeal closure.

66
Q

The internal carotid artery supplies which of the following arteries of the scalp?

(A) Frontalis
(B) Occipitalis
(C) Parietal
(D) Posterior auricular
(E) Supraorbital

A

The correct response is Option E.

The supraorbital and supratrochlear arteries are branches of the internal carotid artery via the ophthalmic artery and, therefore, receive their blood supply from the internal carotid. The arteries of the scalp travel through the subcutaneous fat from the periphery toward the vertex, then anastomose in the midline with branches of the ophthalmic artery. If the internal carotid artery is thrombosed, branches of the external carotid arteries supply blood to the anterior part of the scalp through the angular and ophthalmic arteries.

The frontalis and parietal arteries are branches of the superficial temporal artery, which in turn branches from the external carotid arteries. The occipitalis and posterior auricular arteries are also branches of the external carotid arteries.

67
Q

The levator palpebrae superioris muscle originates from which of the following structures?

(A) Annulus tendineus
(B) Bulbar fascia
(C) Ethmoid bone
(D) Frontal bone
(E) Sphenoid bone

A

The correct response is Option E.

The seven ocular muscles in the orbit are the levator palpebrae superioris, the superior and inferior oblique, and the superior, inferior, medial, and lateral recti. The rectus muscles and the superior oblique arise from the fibrous ring around the optic nerve (the annulus tendineus communis). In contrast, the inferior oblique arises from the maxilla, while the levator palpebrae superioris arises from the lesser wing of the sphenoid bone.

68
Q

Which of the following structures is derived from the second branchial arch?

(A) Incus
(B) Malleus
(C) Sphenoid (greater wing)
(D) Stapes
(E) Temporal bone (squamous portion)

A

The correct response is Option D.

The stapes is derived from Reichert’s cartilage, which is the precursor of the second branchial arch. Other structures derived from the second branchial arch include the stylohyoid process, the lesser cornu of the hyoid bone, and the upper part of the body of the hyoid bone.

The other two ossicles of the middle ear (malleus and incus) are derived from the first branchial arch. Cartilage precursors in the first branchial arch are the quadrate cartilage and Meckel’s cartilage, both of which give rise to skeletal elements through endochondral ossification. The quadrate cartilage gives rise to the greater wing of the sphenoid bone and the incus, while Meckel’s cartilage gives rise to the malleus and the mandibular condyles.

The skeletal elements of the maxillary and mandibular prominences originate from intramembranous ossification, which is derived from direct ossification of the dermal mesenchyme of the arches. The maxillary prominence gives rise to the maxilla, zygoma, and squamous portion of the temporal bone. The mandibular prominence gives rise to the body and ramus of the mandible.

69
Q

Which of the following structures drains into the inferior meatus?

(A) Anterior ethmoid air cells
(B) Frontal sinus
(C) Maxillary sinus
(D) Nasolacrimal duct
(E) Sphenoid sinus

A

The correct response is Option D.

Knowledge of the anatomy of the sinuses is important in the management of patients who have fractures of the frontal sinus. Patency of the drainage system, including the frontonasal and nasolacrimal ducts, must be assessed, as occlusion of the ducts may result in the development of infection or mucocele.

The nasolacrimal duct drains into the inferior meatus. In contrast, the anterior ethmoid air cells and maxillary sinus drain into the middle meatus; the frontal sinus also drains into the middle meatus via the frontonasal duct. The sphenoid sinus drains into the sphenoethmoid recess, which is found above and behind the superior concha.

70
Q

A patient sustains a laceration of the superior helix of the auricle. Adequate anesthesia is most likely to be obtained with block of which of the following nerves?

(A) Facial (VII)
(B) Glossopharyngeal (IX)
(C) Great auricular
(D) Trigeminal (V)
(E) Vagus (X)

A

Superior auricle is innervated by the auriculotemporal nerve, a branch of the mandibular branch of trigeminal (CN V3)

71
Q

Which of the following anatomic structures of the ear originates from the second (hyoid) pharyngeal arch?

(A) Antitragus
(B) Helical root
(C) Superior helix
(D) Tragus

A

The correct response is Option A.

During the sixth week of fetal gestation, the anatomic subunits of the auricle arise from six hillocks that are derived from the first and second pharyngeal arches. The anterior three hillocks from the first (or mandibular) pharyngeal arch ultimately develop into the tragus, helical root, and superior helix. The antihelix, antitragus, inferior helix, and lobule form from the fourth through sixth posterior hillocks from the second (or hyoid) pharyngeal arch. The structures that arise from the first pharyngeal arch typically drain into the parotid lymph nodes, and structures from the second pharyngeal arch drain into the cervical lymph nodes.

72
Q

During dissection to the level of the digastric tendon in a patient undergoing microsurgical head and neck reconstruction, the hypoglossal nerve can be found in which of the following positions?

(A) Lateral to both the internal and external carotid arteries
(B) Medial to both the internal and external carotid arteries
(C) Lateral to the internal carotid artery and medial to the external carotid artery
(D) Medial to the internal carotid artery and lateral to the external carotid artery

A

The correct response is Option A.

During preparation of recipient vessels for free tissue transfer in a patient undergoing head and neck reconstruction, the branches of the carotid artery and internal jugular vein are dissected. The digastric tendon is divided or retracted, and then the carotid artery and hypoglossal nerve are exposed. The hypoglossal nerve passes between the internal carotid artery and internal jugular vein. It turns anteriorly, spiraling around the internal carotid artery, and passes under the occipital artery. It then loops across the lateral surface of the external carotid artery and passes deep to the insertion of the stylohyoid muscle and digastric sling before disappearing beneath the posterior edge of the mylohyoid muscle. The surgeon must take great care to avoid injuring the hypoglossal nerve because injury would lead to impaired tongue mobility and potential difficulties with speech and swallowing.

73
Q

Intracranial communication of a frontonasal encephalocele is most likely to occur through which of the following anatomic structures?

(A) Cribriform plate
(B) Foramen rotundum
(C) Foramen ovale
(D) Foramen cecum
(E) Superior orbital fissure

A

The correct response is Option D.

Intracranial communication of a frontonasal encephalocele is most likely to occur through the foramen cecum because anterior encephaloceles herniate through this foramen. In contrast, the cribriform plate transmits the fibers of the olfactory nerve, the foramen rotundum transmits the maxillary division of the trigeminal nerve (V2), and the foramen ovale transmits the mandibular division of the trigeminal nerve (V3). The superior orbital fissure transmits the oculomotor (III), trochlear (IV), and abducens (VI) nerves, as well as the ophthalmic division of the trigeminal nerve (V1).

74
Q

In a patient who has undergone Le Fort I osteotomy, which of the following arteries provides the primary blood supply to the maxilla?

(A) Ascending pharyngeal artery
(B) Greater palatine artery
(C) Infraorbital artery
(D) Lesser palatine artery
(E) Posterior superior alveolar artery

A

The correct response is Option A.

Although the maxilla has an extensive blood supply as a result of its multiple anastomotic connections, much of its vascularity is irreversibly severed during the incisions and mobilization required for the Le Fort I osteotomy. Prior to surgery, the internal maxillary artery supplies blood to the maxilla through the descending palatine, posterior superior alveolar, and infraorbital arteries. However, after a Le Fort I osteotomy is performed, the ascending palatine branch of the facial artery and the palatine branch of the ascending pharyngeal artery provide the primary vascular supply. Both vessels are branches of the external carotid artery and contribute somewhat to the maxillary blood supply prior to surgery.

The greater palatine artery emerges from the greater palatine foramen and courses anteriorly; its arterial branches are distributed to the palate and soft tissue of the roof of the mouth. The lesser palatine artery emerges from the lesser palatine foramen and supplies vascularity to the soft palate and palatine tonsils.

75
Q

During a rhytidectomy procedure, the risk for injury to the great auricular nerve is greatest at which of the following locations?

(A) 1 cm anterior to the tragus
(B) 2 cm posterior to the lobule
(C) 4 cm posterior to the lobule
(D) 6 cm inferior to the tragus
(E) 10 cm inferior to the tragus

A

The correct response is Option D.

The great auricular nerve emerges from behind the sternocleidomastoid muscle 9 cm below the caudal edge of the external auditory canal and 6 cm inferior to the tragus. The nerve lies posterior and superficial to the submuscular aponeurotic system (SMAS) and platysma at this point and is at greatest risk for injury during rhytidectomy. Injury to this nerve can result in numbness or painful dysesthesias of the lower two thirds of the ear and the adjacent skin of the neck and cheek.

76
Q

The external acoustic meatus is derived from which of the following structures?

(A) First branchial arch
(B) Second branchial arch
(C) First branchial groove
(D) Second branchial groove
(E) Third branchial groove

A

The correct response is Option C.

The external acoustic meatus is derived from the first branchial groove. During fetal development, the external ear arises from six hillocks derived from the first and second branchial arches. The anterior three hillocks (from the first branchial arch) form the anterior portion of the auricle, and the posterior auricle is formed from the fourth through sixth posterior hillocks (from the second branchial arch). The second through fourth branchial grooves are typically obliterated within the cervical sinus by the end of the seventh week of gestation, resulting in a neck with a smooth contour; however, a persistent branchial groove can manifest as a branchial fistula, sinus, or cyst.

77
Q

Which of the following permanent teeth erupts first?

(A) Central incisor
(B) Lateral incisor
(C) Canine
(D) First premolar
(E) First molar

A

The correct response is Option E.

The first molar is the first permanent tooth to erupt; this typically occurs between ages 6 and 7 years. The age of mixed dentition, in which there are both deciduous (primary) and permanent (secondary) teeth erupted in the oral cavity simultaneously, begins with the eruption of the first molars. In addition, Angle’s classification, which describes the relationship between the mesiobuccal cusp of the maxillary first molar and the buccal groove of the mandibular first molar, has its basis in this pattern of eruption.

Knowledge of the eruption pattern of the teeth is crucial for management of facial fractures in children, especially for coordination of any necessary bone grafting and/or orthognathic surgery.

The central incisors erupt between ages 6 and 8 years, the lateral incisors between ages 7 and 9 years, the canine teeth between ages 9 and 12 years, and the first premolars between ages 10 and 12 years.

78
Q

In addition to the zygoma, which of the following bones forms the lateral orbital wall?

(A) Frontal bone
(B) Greater wing of the sphenoid
(C) Lacrimal bone
(D) Lesser wing of the sphenoid
(E) Maxilla

A

The correct response is Option B.

The lateral orbital wall is formed primarily by the orbital surface of the zygomatic bone and the greater wing of the sphenoid bone. In addition to the greater wing of the sphenoid, other bones that comprise the orbit include the ethmoid, frontal bone, lacrimal bone, maxilla, palatine bone, and the lesser wing of the sphenoid. The sphenoid portion of the lateral wall is separated from the roof of the orbit by the superior orbital fissure and from the floor of the orbit by the inferior orbital fissure.

Appropriate anatomic reduction of the greater wing of the sphenoid and lateral wall of the orbit is critical for adequate reduction of a fracture of the zygomaticomaxillary complex. Because correct reduction of the inferior orbital rim, zygomaticofrontal suture, and orbital floor does not ensure reduction of the lateral wall, the surgeon should verify that the complete reduction of all structures has been performed.

79
Q

Which of the following structures drains into the middle meatus?

(A) Frontal sinus
(B) Mastoid air cells
(C) Nasolacrimal duct
(D) Posterior ethmoidal air cells
(E) Sphenoid sinus

A

The correct response is Option A.

The frontal sinus drains into the middle meatus via the frontonasal duct. In addition, the anterior ethmoidal air cells and maxillary sinus drain into the middle meatus. It is important to assess the patency of this drainage system when managing fractures of the frontal sinus because any occlusion of the frontonasal duct may result in infection or mucocele.

The mastoid air cells communicate with the mastoid antrum, which then communicates with the tympanic cavity. The nasolacrimal duct drains into the inferior meatus, and the posterior ethmoidal air cells drain into the superior meatus. The sphenoid sinus drains into the sphenoethmoid recess, which is located above and behind the superior concha.

80
Q

Which of the following best describes the primary action of the superior oblique muscle on the globe?

(A) Abduction
(B) Adduction
(C) Depression
(D) Elevation
(E) Extorsion

A

The correct response is Option C.

The superior oblique muscle primarily acts to depress the globe. Its secondary actions include abduction and intorsion. This muscle originates outside of and superomedial to the annulus of Zinn (the common tendinous ring at the orbital apex). It courses around the trochlear nerve to insert onto the globe. Sensory innervation to the superior oblique muscle is provided by the trochlear nerve.

81
Q

Which of the following best describes the blood supply to the sternocleidomastoid muscle?

(A) One source
(B) One dominant source and one minor source
(C) Two equally dominant sources
(D) Three equally dominant sources
(E) Four equally dominant sources

A

The correct response is Option D.

The sternocleidomastoid muscle attaches to the mastoid process superiorly and to the clavicle and sternum inferiorly. The blood supply to this muscle is segmental and is derived from three equally dominant sources, each perfusing a portion of the muscle with some internal connections; each portion has its own musculocutaneous perforators that supply a small area of overlying skin. Vascularity to the superior third of the muscle is supplied by a branch of the occipital artery, to the middle third by a branch of the superior thyroid artery, and to the inferior third by a branch of the thyrocervical trunk.

Knowledge of this pattern of arterial anatomy is important when harvesting the muscle for reconstruction of the neck and mandible. If the entire muscle is harvested for use but is based only on the inferior or superior pedicle, the portion of muscle farthest from the pedicle may not be reliable.

82
Q

Which of the following structures does NOT attach to the lateral orbital tubercle?

(A) Fascial extension of the lateral rectus muscle
(B) Lateral check ligament
(C) Lateral horn of the levator aponeurosis
(D) Lateral limb of Lockwood’s ligament
(E) Lateral palpebral ligament

A

The correct response is Option D.

Attachments to the lateral orbital tubercle (also referred to as Whitnall’s tubercle), which is found within the zygomatic bone approximately 10 mm beneath the lateral orbital rim, include the fascial extension of the lateral rectus muscle, the lateral check ligament, the lateral horn of the levator aponeurosis, and the lateral palpebral ligament, which anchors the tarsal plates.

Although the suspensory ligament of Lockwood attaches to the walls of the orbit and contributes to the thickened lower portion of the bulbar sheath, it does not attach to the lateral orbital tubercle. This contribution to the bulbar sheath is formed by fascia from the inferior rectus and inferior oblique muscles as they cross beneath the globe.

83
Q

The deep division of the supraorbital nerve provides sensation to which of the following areas?

(A) Central forehead
(B) Central scalp
(C) Nasal radix
(D) Temporal forehead
(E) Temporal scalp

A

The correct response is Option B.

Knowledge of the anatomy of the supraorbital nerve is crucial for minimizing the risk for nerve injury and subsequent numbness during endoscopic forehead lifting. The supraorbital nerve is a branch of the ophthalmic division of the trigeminal nerve (V1). This nerve arises from a foramen or notch along the superior orbital rim, and then divides immediately into deep and superficial branches. The deep division courses laterally toward the superior temporal line of the skull, and then continues to the coronal suture to supply sensation to the central frontoparietal scalp. In contrast, the superficial branch divides into multiple branches, each of which courses cephalad into the frontalis muscle to supply sensation to the central forehead and hairline.

Sensation to the nasal radix is supplied by the supratrochlear and infratrochlear nerves, while the maxillary and mandibular branches of the trigeminal nerve (V2 and V3) supply sensation to the temporal forehead. Sensation to the temporal scalp is supplied by the occipital nerve.

84
Q

The palatine bone contributes to which of the following structures?

(A) Anterior nasal spine
(B) Dental arch
(C) External nose
(D) Hamulus
(E) Orbit

A

The correct response is Option E.

The palatine bone is located posterior to the maxilla and pterygoid lamina; it is composed of horizontal and pyramidal processes. The horizontal process contributes to the posterior aspect of the hard palate and becomes the floor of the choana. The pyramidal process extends vertically to contribute to the floor of the orbit.

The anterior nasal spine is a prominence of the maxillary bone.

The dental arch lies anterior to the palatine bone because it includes a portion of the maxilla. The hamulus extends inferiorly from the medial pterygoid plate, posterior to the palatine bone.

The external nose is comprised of the paired nasal bones and frontal processes of the maxilla.

85
Q

An 8-year-old boy has had a painless mass in the midline of the neck that has slowly enlarged over the past two years. This finding is most consistent with

(A) branchial cleft cyst
(B) dermoid cyst
(C) laryngocele
(D) lymphatic malformation
(E) thyroglossal duct cyst

A

The correct response is Option E.

This 8-year-old boy has a thyroglossal duct cyst, a slowly enlarging, painless mass of the midline of the neck that results from incomplete degeneration of the thyroglossal duct before birth. This duct connects an endodermal diverticulum (which later becomes the thyroid gland) to the foramen cecum in the developing fetus; in most cases, the duct degenerates once the diverticulum becomes a solid gland. Although thyroglossal duct cysts can be found from the base of the tongue to the cricoid cartilage, most cysts are located inferior to the hyoid bone. Infection and rupture are fairly uncommon; if these occur, antibiotics should be administered. Definitive management of a thyroglossal duct cyst is complete surgical excision of the cyst and central portion of the hyoid bone; this is known as the Sistrunk procedure.

Branchial cleft cysts rarely occur in the midline of the neck. Instead, most of these cysts develop from remnants of the second branchial cleft and are found at the anterior border of the sternocleidomastoid muscle. Dermoid cysts are also rare in the neck and are more likely to affect the nasion, lateral brow, or calvarium. A laryngocele is an air pocket that normally communicates with the larynx and often enlarges with coughing or other changes in pressure within the trachea; the report of a slowly enlarging, painless mass would not be expected to describe a laryngocele. Lymphatic malformations are present at birth and would not be seen in the midline of the neck. Because these malformations do not occur in a specific plane, diffuse involvement of head and neck tissues is common.

86
Q

Patients with paralysis of the trigeminal nerve have loss of function of which of the following muscles?

(A) Levator veli palatini
(B) Palatopharyngeus
(C) Stylohyoid
(D) Stylopharyngeus
(E) Tensor veli palatini

A

The correct response is Option E.

Because the tensor veli palatini muscle receives its motor innervation from the trigeminal (V) nerve, a patient who has paralysis of this nerve will experience a loss of function of the tensor veli palatini, which is a derivative of the first branchial arch. In contrast, the anatomically adjacent levator veli palatini muscle is a derivative of the fourth branchial arch and is innervated by the superior laryngeal branch of the vagus (X) nerve. The palatopharyngeus muscle has this same origin and innervation. The stylohyoid muscle is derived from the second branchial arch and innervated by the facial (VII) nerve, while the stylopharyngeus muscle is derived from the third branchial arch and innervated by the glossopharyngeal (IX) nerve.

87
Q

A 55-year-old woman has numbness of the earlobe after undergoing biopsy of an internal jugular lymph node. The most likely cause is injury to which of the following structures?

(A) Auricular branch of the vagus nerve
(B) Auriculotemporal nerve
(C) Great auricular nerve
(D) Posterior auricular nerve
(E) Ventral ramus of the first cervical root

A

The correct response is Option C.

This 55-year-old woman has numbness of the earlobe due to injury to the great auricular nerve. This nerve arises from the second and third cervical nerves and emerges from the posterior border of the sternocleidomastoid muscle, then travels anterosuperiorly between the sternocleidomastoid and platysma muscles and divides into auricular, facial, and mastoid branches. The auricular branch provides sensation to the earlobe and posterior two thirds of the ear and is prone to injury during surgery in the region of the upper lateral neck.

The auricular branch of the vagus nerve, also known as Arnold’s nerve, arises from the superior ganglion, receives a contribution from the glossopharyngeal nerve, and travels along the temporal bone, emerging through the auricular fissure between the mastoid process and external auditory meatus. It supplies sensation to the posterior aspect of the ear and external auditory meatus.

The auriculotemporal nerve is a sensory branch of the trigeminal nerve. This nerve branch courses posterior to the external pterygoid muscle, emerges from beneath the parotid gland, crosses the zygoma, and travels in a superior direction. The anterior auricular branches of this nerve supply sensory innervation to the anterior upper helix and tragus.

The posterior auricular nerve arises from the facial nerve at the stylomastoid foramen, receives a contribution from the auricular branch of the vagus nerve, and supplies two nerve branches, one of which joins with the mastoid branch of the great auricular nerve and another that joins with the lesser occipital nerve. Both branches provide sensibility to the posterior side of the pinna and the concha. The posterior auricular nerve supplies motor innervation to the posterior auricular and occipitalis muscles.

The ventral ramus of the first cervical root courses parallel to the vertebral artery and supplies motor innervation to the rectus capitis muscles.

88
Q

Which of the following structures is a branch of the mandibular division of the trigeminal nerve?

(A) Infraorbital nerve
(B) Lingual nerve
(C) Nasopalatine nerve
(D) Posterosuperior alveolar nerve
(E) Posterosuperior nasal nerve

A

The correct response is Option B.

The lingual nerve, which supplies sensation to the anterior two thirds of the tongue, is a branch of the mandibular division of the trigeminal nerve (V3). Other structures that arise from this nerve include the inferior alveolar nerve, which supplies sensation to the mandibular teeth; the long buccal branch, which supplies sensation to the buccal mucosa; and the mental nerve, which supplies sensation to the skin of the chin and lower lip and the mucosa of the lip and adjacent gingiva. In addition, the auriculotemporal nerve divides from the posterior border of V3 immediately after exiting the foramen ovale, passes around the middle meningeal artery as two units, and then courses between the external auditory canal and temporomandibular joint (TMJ). This nerve supplies sensory innervation to the anterior auricle, a large portion of the temporal region, and part of the external auditory canal and gives off a branch to supply the TMJ.

The infraorbital nerve, nasopalatine nerve, posterosuperior alveolar nerve, and posterosuperior nasal nerve are branches of the maxillary division of the trigeminal nerve (V2).

89
Q

A 42-year-old man is scheduled to undergo surgical excision of a lesion of the lower lip. During anesthetic blockade of the mental nerve prior to the procedure, the nerve foramen can be located beneath the apex of which of the following mandibular teeth?

(A) Central incisor
(B) Cuspid
(C) First molar
(D) Lateral incisor
(E) Second bicuspid

A

The correct response is Option E.

During anesthetic blockade of the mental nerve, the nerve foramen can be found beneath the apex of the second bicuspid tooth. This nerve, which is the terminal branch of the inferior alveolar nerve, supplies sensory innervation to the skin and mucous membranes of the lower lip as well as the skin of the anterior mandible and chin.

It is important to have knowledge of the location of the mental nerve and its foramen in order to prevent injury during reduction of anterior mandibular fractures or osseous sliding genioplasty procedures.

Because the inferior alveolar nerve is a branch of the mandibular division of the trigeminal nerve (V3), it may be injured during sagittal split osteotomy procedures. Affected patients will have numbness in the distribution of the inferior alveolar nerve postoperatively.

90
Q

Stensen’s duct can be found at which of the following anatomic sites?

(A) At the mandibular angle
(B) At the preauricular border
(C) At the zygomatic arch
(D) Between the superficial and deep lobes of the parotid gland
(E) Within the buccal space

A

The correct response is Option E.

Stensen’s duct can be found within the buccal space, which is bordered anteriorly by the orbicularis oris muscle, posteriorly by the edge of the masseter muscle, superiorly by the zygomaticus major muscle, and inferiorly by the fascial attachment of the buccinator muscle to the mandible. The duct develops deep within the parotid gland and emerges from the superior third of the gland at its anterior border, then courses below the zygomatic arch and enters the buccal space, inserting into the buccinator and then entering the oral cavity opposite the upper second molar. The facial artery and vein, buccal branches of the facial nerve, and buccal fat pad can also be found within the buccal space.

Although Stensen’s duct and branches of the facial nerve are vulnerable to inadvertent dissection in the area of the parotid gland, there are no major arterial branches in this region.

The external carotid artery can be found within the angle of the mandible, while the superficial temporal artery is located within the preauricular border. The transverse facial artery is found at the zygomatic arch. These locations do not lie within the typical course of Stensen’s duct.

91
Q

Each of the following is a muscle of mastication EXCEPT the

(A) buccinator
(B) lateral pterygoid
(C) masseter
(D) medial pterygoid
(E) temporalis

A

The correct response is Option A.

The lateral and medial pterygoids, masseter, and temporalis muscles are muscles of mastication, capable of exerting force on the mandible. These muscles are innervated by the mandibular division of the trigeminal nerve (V3).

The lateral pterygoid muscle has two heads, which arise from the infratemporal surface and infratemporal crest of the greater wing of the sphenoid bone and the lateral surface of the lateral pterygoid plate, and insert on the mandibular neck and the articular capsule and disk of the temporomandibular joint. This muscle pulls the condylar process of the mandible and articular disk forward, opening the mouth.

The medial pterygoid muscle arises from the medial surface of the lateral pterygoid plate and the pyramidal process of the palatine bone and inserts on the medial surface of the mandibular ramus and angle. It acts to elevate and protrude the mandible and produce side-to-side movements.

The masseter muscle arises from the zygomatic arch and inserts on the mandibular ramus and coronoid process; during mastication it elevates the mandible to occlude the teeth.

The temporalis muscle arises from the temporal fossa, passes deep to the zygomatic arch, and inserts on the coronoid process and anterior border of the mandibular ramus. This muscle elevates the mandible and contributes to side-to-side grinding movements. The posterior fibers of the temporalis aid in retraction of the mandible once it has been protruded.

The buccinator muscle arises from the alveolar process of the maxilla, from the mandible opposite the molars, and from the anterior border of the pterygoid mandibular raphe; it inserts into the submucosa of the cheeks and lips and is used to compress the cheeks against the teeth and gums. It is innervated by the facial (VII) nerve. Although it is not a muscle of mastication, it assists the tongue in directing food between the molars during mastication.

92
Q

Which of the following provides the drainage ostium for the nasolacrimal duct into the nose?

(A) Inferior meatus
(B) Middle meatus
(C) Superior meatus
(D) Supreme meatus

A

The correct response is Option A.

The inferior meatus provides the drainage ostium for the nasolacrimal duct into the nose. The maxillary, frontal, and anterior ethmoidal sinuses drain into the middle meatus, while the posterior ethmoid cells drain into the superior meatus. The sphenoid sinus drains into the supreme meatus, also referred to as the sphenoethmoidal recess.